Craniofacial Flashcards

1
Q

A 20-year-old woman is evaluated for facial asymmetry due to loss of volume on one side of the face. She has no history of trauma or infection to that side of her face. Physical examination shows normal dental occlusion with normal bony anatomy and no cranial nerve deficits. She has a mild-to-moderate deficiency of soft tissue along the cheek between the zygomatic and mandibular areas of the face. Which of the following is the most appropriate intervention for correction of this patient’s hemifacial atrophy?

A) Adipofascial free flap
B) Autologous fat grafting
C) Distraction osteogenesis
D) Hyaluronic acid filler
E) Silicone injection

A

The correct response is Option B.

Romberg disease, also known as progressive hemifacial atrophy (PHA), is an acquired disease causing severe deformity of the facial contour. The therapeutic goals of surgery for patients with PHA are to minimize psychosocial effects and to correct the appearance and function of involved facial structures. PHA usually presents in the first 20 years of life, after which it is slowly progressive but self-limited. The atrophy affects subcutaneous tissue, fat, muscle, and osteocartilaginous structures, creating a sunken hemiface appearance. Epidermal cutaneous involvement is minimal, but the tongue, gingiva, teeth, and palate may also be involved. For cases with minimal-to-moderate soft-tissue defects, autologous fat grafting can provide correction of the contour deformities. For more severe soft-tissue deficits, additional or other modalities such as adipofascial free flaps can be used. In more severe cases with skeletal and/or dental abnormalities, interventions such as distraction osteogenesis and bone grafting may be necessary. The injection of free silicone is not recommended and has been replaced by the use of autologous fat and other injectable substances. Hyaluronic acid may provide a temporary solution, but it is costly.

2018

How well did you know this?
1
Not at all
2
3
4
5
Perfectly
2
Q

The patient shown in the photograph is most likely to have which of the following comorbidities?

A) Anosmia
B) Craniosynostosis
C) Epiphora
D) Sensorineural hearing loss
E) Velopharyngeal dysfunction

A

The correct response is Option C.

The patient pictured has a rare craniofacial (Tessier No. 4) cleft. Rare craniofacial clefts are characterized by variable soft-tissue and bony involvement. This patient has a cleft lip that begins lateral to the philtral column and courses laterally to the alar margin. Additionally, this patient has soft-tissue deficiency of the left medial malar region, as well as medial lower lid malposition and medial canthal dystopia. Of all of the options given, these findings suggest an increased risk for nasolacrimal duct abnormalities or inferior canalicular system malposition. Velopharyngeal dysfunction is common in patients with cleft lip and palate. Anosmia can be present in midline craniofacial anomalies, such as craniofrontonasal dysplasia. Rare craniofacial (Tessier No. 4) clefts are not associated with sensorineural hearing loss or craniosynostosis.

How well did you know this?
1
Not at all
2
3
4
5
Perfectly
3
Q

A 3-year-old boy is brought for evaluation of a right frontotemporal cranial defect. A photograph is shown. Which of the following materials is most appropriate for reconstruction?

A) Autologous bone
B) Bone morphogenetic protein (BMP)
C) Calcium phosphate paste
D) Porous polyethylene
E) Titanium mesh

A

The correct response is Option A.

Autologous bone is the most appropriate material for inlay cranioplasty for a child this age. Each of the other options has been used in the pediatric population, but they carry important limitations. Porous polyethylene can be used to reconstruct large cranial defects, but is expensive to fabricate, has the potential to become unstable in a growing cranium, and has a higher overall infection rate than autologous bone. Bone morphogenetic protein (BMP) has shown promise in animal models for inducing bone formation in large cranial defects, but it is expensive for a defect of this size; it is a powerful mitogen and can result in dural ossification and, potentially, induction of tumor growth. Calcium phosphate pastes also have been advocated but are very expensive and for a defect of this size, the long-term potential osseointegration is dubious. Furthermore, the risk of infection and/or implant extrusion is higher. Titanium mesh may be appropriate for some children with limited life expectancy or functional needs, but these implants can be bent, offer limited impact resistance, and have a higher overall rate of complications than autologous bone.

2018

How well did you know this?
1
Not at all
2
3
4
5
Perfectly
4
Q

A newborn male is evaluated in the office for cleft of the soft palate. Physical examination shows severe micrognathia with labored breathing, underdevelopment of the mid face, microtia, and coloboma of the lower eyelids. Mutation in which of the following genes most likely caused these findings?

A) EFNB1
B) FGFR2
C) FGFR3
D) TCOF1
E) TWIST1

A

The correct response is Option D.

Treacher Collins syndrome, named after Edward Treacher Collins, the English ophthalmologist who first described this condition in 1900, is characterized by widely varying degrees of micrognathia, underdevelopment and/or clefting of the midfacial bones (e.g., zygoma), microtia or anotia with or without hearing loss or middle ear atresia, eyelid coloboma, and often respiratory compromise. It occurs with an estimated frequency of 1:50,000 live births, and the overwhelming majority of cases are caused by an autosomally inherited mutation in the TCOF1 gene that leads to underproduction of treacle protein, a neural crest precursor.

Mutations in FGFR2, FGFR3, TWIST1, and EFNB genes are commonly associated with syndromic forms of craniosynostosis. Gain-of-function mutations in the fibroblast growth factor receptors (FGFR) can lead to various syndromes that manifest craniosynostosis including: Apert syndrome (FGFR2), Pfeiffer syndrome (FGFR1 and FGFR2), Crouzon syndrome (FGFR2), Jackson–Weiss syndrome (FGFR2), Muenke syndrome (FGFR3), Crouzonodermoskeletal syndrome (FGFR3), and thanatophoric dysplasia (FGFR3). TWIST1 loss-of-function mutations can lead to another craniosynostosis syndrome—Saethre-Chotzen syndrome. EFNB1 mutations are associated with craniofrontonasal syndrome, an X-linked disorder characterized by hypertelorism, craniosynostosis, clefting of the nasal tip, and digital anomalies.

2018

How well did you know this?
1
Not at all
2
3
4
5
Perfectly
5
Q

Which of the following is the protein encoded by the gene mutated in van der Woude syndrome and popliteal pterygium syndrome?

A) Fibroblast growth factor 10 (FGF10)
B) Fibroblast growth factor receptor 2b (FGFR2b)
C) Interferon regulatory factor 6 (IRF6)
D) Membrane-bound signaling molecule jagged 2 (JAG2)
E) Sonic hedgehog (SHH)

A

The correct response is Option C.

The mechanism of palatal shelf formation must be regulated precisely so that vertical palatal shelves are adhesion-incompetent while they are in close contact with other structures, but once they are raised above the tongue, they rapidly acquire adhesion capability if they are not to remain cleft. Control of periderm differentiation by the membrane-bound signalling molecule jagged 2 (JAG2) is important in this process. Another factor central to this process is interferon regulatory factor 6 (IRF6). The gene IRF6, which has a causal association with van der Woude syndrome, is also linked strongly to the isolated form of clefting. This finding has been replicated in many different populations and ethnic groups. Variants of genes linked to syndromic forms of cleft lip with or without cleft palate that have a mendelian mode of inheritance can also produce phenocopies of non-syndromic clefts. This observation suggests that a strategy of choosing variants of genes associated with syndromic forms of cleft lip with or without cleft palate as candidates for investigations into the cause of non-syndromic clefts could be productive. Other examples of mendelian-inherited syndromes and related genes that, if mutated, could result in or modify the expression of cleft lip with or without cleft palate include Kallmann syndrome (FGFR1), ectrodactyly, ectodermal dysplasia, and clefting syndrome (TP63), X-linked clefting and ankyloglossia (TBX22), Gorlin syndrome (PTCH1), and Margarita Island ectodermal dysplasia (PVRL1[heterozygous]). Although discovery of the genetic cause of van der Woude or popliteal pterygium syndromes will have no immediate therapeutic benefit, advantages for diagnosis are instant, and this knowledge will be potentially useful in genetic counseling. If one gene mutation, which can be identified by prenatal diagnosis, causes cleft lip, cleft lip and palate, or cleft palate alone in a proportion of people, identification of individuals at high risk for having children with the same defect will be possible.

2018

How well did you know this?
1
Not at all
2
3
4
5
Perfectly
6
Q

An otherwise healthy 5-year-old girl has a yellow, cystic mass on the left sclera, a left preauricular branchial cleft remnant, elevation of the left oral commissure, and soft-tissue deficiency of the left face. It is most appropriate to order which of the following tests to assess for significant comorbidities?

A) Echocardiography
B) Fluorescence in situ hybridization of the branchial cleft remnant
C) Funduscopic examination
D) MRI of the brain
E) Plain x-ray studies of the hands

A

The correct response is Option C.

The patient described has Goldenhar syndrome, a severe form of hemifacial microsomia with variable ear anomalies, mandibular hypoplasia leading to occlusal cant and oral commissure asymmetry, and soft-tissue deficiency on the affected side. Ocular findings associated with Goldenhar syndrome are variable and common (50% incidence) and can occur as epibulbar dermoids as described in this patient as well as microphthalmia, eyelid and optic nerve colobomas (interruption of a circular structure of the eye). Colobomas can be asymptomatic (affecting the iris alone), can lead to exposure keratopathy (in the case of eyelid colobomas), or can lead to visual disturbances (optic disc/nerve coloboma), such as visual field deficits or amblyopia. Care must be taken to identify visual disturbances early in order to implement contralateral eye penalization, or patching, to maximize visual development of the affected eye. The other tests target areas that are not affected in patients with Goldenhar syndrome.

2018

How well did you know this?
1
Not at all
2
3
4
5
Perfectly
7
Q

A 4-year-old boy is brought to the office for evaluation of the shape of his head. Physical examination shows an asymmetrically shaped head, low frontal hairline, ptosis of the eyelids, and a prominent crus of helix. Which of the following syndromes is most likely affecting this child?

A) Apert
B) Crouzon
C) Muenke
D) Pfeiffer
E) Saethre-Chotzen

A

The correct response is Option E.

Saethre-Chotzen syndrome is an autosomal dominant disorder associated with TWIST1 gene mutation. It causes an asymmetrical brachycephaly, bicoronal or unicoronal synostosis, low frontal hairline, ptosis of the eyelids, and a prominent crus of helix.

Crouzon syndrome is an autosomal dominant disorder associated with an FGFR2 gene mutation. The patient with this syndrome typically comes to the office because of hydrocephalus, elevated intracranial pressure, Chiari I malformation, bicoronal synostosis and brachycephaly, exorbitism, mid-face hypoplasia, anterior open bite, and normal extremities.

Apert syndrome is an autosomal dominant disorder and shows symptoms of elevated intracranial pressure, bicoronal synostoses, turribrachycephaly, enlarged anterior fontanel, bitemporal widening, occipital flattening, anterior open bite, and complex syndactyly of the hands and feet.

Pfeiffer syndrome is also an autosomal dominant disorder associated with FGFR2 mutation and reveals symptoms of hydrocephalus and a high risk of Chiari malformation, turribrachycephaly with bicoronal synostoses, exorbitism, hypertelorism, down-slanting palpebral fissures, an anterior open bite, and broad thumbs and/or broad halluces.

Muenke syndrome is associated with the FGFR3 gene mutation and comes to the office with coronal synostosis, elevated intracranial pressure, sensorineural hearing loss, and abnormal middle phalanges.

2018

How well did you know this?
1
Not at all
2
3
4
5
Perfectly
8
Q

A 9-year-old boy with Crouzon syndrome is brought to the craniofacial clinic because of obstructive sleep apnea. Apnea hypopnea index is 40. He has a negative overjet of 12 mm, moderate exorbitism, and signs of increased intracranial pressure (>25 mm Hg), which was found during an intracranial pressure monitoring trial last month; intercanthal distance is normal (25 mm). Which of the following types of osteotomy is the most appropriate treatment for this patient?

A) Facial bipartition
B) Le Fort I
C) Le Fort III
D) Monobloc
E) Orbital box

A

The correct response is Option D.

Crouzon-related obstructive sleep apnea and increased intracranial pressure are best treated with frontofacial advancement without adjustment of the orbital width, which is best achieved using the monobloc osteotomy. Orbital box osteotomy will not treat increased intracranial pressure or sleep apnea. Le Fort III osteotomy will treat obstructive sleep apnea but will minimally affect increased intracranial pressure. Lastly, facial bipartition osteotomy is not needed in this case because of the normal intercanthal distance and lack of telorbitism. Le Fort I addresses neither the orbits nor the intracranial pressure.

2018

How well did you know this?
1
Not at all
2
3
4
5
Perfectly
9
Q

A 25-year-old woman comes to the office because of a 5-year history of stable facial asymmetry. Physical examination shows fullness of the right mandibular angle; no facial tenderness, masses, or lymphadenopathy is noted. Maxillofacial CT scan shows enlargement of the right masseter muscle compared with the left. Which of the following is the most appropriate first-line treatment for this problem?

A) Fabrication of a bite guard to treat bruxism
B) Injection of botulinum toxin type A into the right masseter muscle
C) Open subtotal myectomy of the right masseter muscle
D) Placement of a left mandibular angle prosthesis
E) Referral to oncology to rule out systemic amyloidosis

A

The correct response is Option B.

The patient described has benign or idiopathic masseter hypertrophy. The classic patient comes to the office because of aesthetic complaints but is otherwise asymptomatic. There are numerous treatments, including partial myotomy, mandibular angle reduction, and contralateral augmentation. Of all the options, the best is botulinum toxin type A injections. These injections are minimally invasive and reversible, and they do not require an anesthetic. On the basis of its low–risk profile, it is the best first-line treatment listed. Amyloidosis is a progressive systemic disease that can cause myopathy and painful bilateral masseter hypertrophy. It is unlikely in this stable, unilateral case. Bruxism can cause masseter hypertrophy, which is most often bilateral. A bite guard will prevent tooth damage, but it will not correct hypertrophy.

2018

How well did you know this?
1
Not at all
2
3
4
5
Perfectly
10
Q

A newborn male is evaluated because of asymmetry of the orbit and eyelid. Physical examination shows a dozen 6- to 10-mm tan macules, axillary freckles, pigmented nodules on the iris, and soft, fleshy, raised, soft-tissue lesions on the forehead and scalp. Which of the following is the most likely diagnosis?

A) Goldenhar syndrome
B) Kabuki syndrome
C) Muenke syndrome
D) Neurofibromatosis
E) Nevus sebaceous

A

The correct response is Option D.

The patient described meets the criteria for neurofibromatosis 1 by clinical diagnosis. Genetic testing is also possible. To make a clinical diagnosis, two or more of the following criteria are required:

Six or more café au lait spots 5 mm or larger (prepubertal) or 15 mm or larger (post-pubertal)

Family history

Two or more neurofibromas

Freckling in armpits or groin

Lisch nodules

Bony dysplasia (sphenoid wing in this case)

Tumor on optic nerve

A patient with Goldenhar syndrome has facial asymmetry, but does not have café au lait spots. Colobomas are common in patients with this syndrome, but Lisch nodules are not.

Muenke syndrome is associated with craniosynostosis. Kabuki syndrome has distinctive associated facial features with long palpebral fissures, everted lower lids, broad nasal tip, arched brows, protruding ears, intellectual disability, and often microcephaly.

Nevus sebaceous is a well-circumscribed hamartomatous lesion composed of sebaceous glands. There is a risk for malignant transformation.

2018

How well did you know this?
1
Not at all
2
3
4
5
Perfectly
11
Q

A 5-year-old boy is referred for evaluation of facial asymmetry. His parents report that they have noticed asymmetry since birth and believed it would improve with time, but it has not. The patient has no history of surgery or physical therapy. On physical examination, the physician notes skull asymmetry and facial asymmetry, including the orbits; there is no obvious neck tilt. A CT scan of the head is most likely to show which of the following?

A) Deformational plagiocephaly
B) Lambdoid craniosynostosis
C) Torticollis
D) Unicoronal craniosynostosis
E) Zygomaticotemporal craniosynostosis

A

The correct response is Option A.

Since the early 1990’s when positional deformational cranial deformities were first identified with the “back-to-sleep” campaigns, they have now become the most common etiology of cranial and facial asymmetry. Prior to that time, torticollis was the most common cause. Craniosynostosis is a much less frequent cause and occurs in the following order of decreasing frequency: unicoronal, lambdoid, zygomaticotemporal. Correct identification of most of the causes of this clinical constellation of findings can be made on clinical examination, but most would still get a CT scan to verify the diagnosis and rule out associated anomalies except for torticollis, which is always a clinical diagnosis.

2018

How well did you know this?
1
Not at all
2
3
4
5
Perfectly
12
Q

A 53-year-old man presents with a calvarial defect following neurosurgical extirpation of a tumor. Polymethylmethacrylate reinforced by wire mesh is used for the cranioplasty. The use of saline irrigation during the curing of the biopolymer serves which of the following purposes?

A) Dilutes toxic byproducts of polymerization
B) Dissipates thermal energy
C) Irrigates subclinical bacterial colonization
D) Provides an aqueous catalyst for polymerization
E) Reduces the dielectric constant of the substrate

A

The correct response is Option B.

Polymethylmethacrylate polymerization is an exergonic or exothermic reaction. The heat generated can cause injury or necrosis of the surrounding soft tissues and bone. Saline irrigation is an effective method of heat dissipation during this process. The toxic byproducts of polymerization include cyanide gas in small amounts that are dissipated by normal operating room ventilation or suction. Water is not a necessary cofactor for the polymerization process. Reduction of bacterial burden is a rationale for irrigation in general but does not play a specific role in this clinical setting. The dielectric constant of the substrate is elevated or unchanged by the irritant and this is unlikely to have an effect on the polymerization process.

2017

How well did you know this?
1
Not at all
2
3
4
5
Perfectly
13
Q

While performing a cranioplasty on a 6-year-old girl, the plastic surgeon accidentally drops the bone graft on the floor. Which of the following is the most appropriate next step?

A) Change the wound classification from 1 to 4
B) Decontaminate with triple antibiotic solution lavage and reuse the graft
C) Discard the graft and use an alloplastic material
D) Steam sterilize and reuse the graft
E) Wash with chlorhexidine and reuse the graft

A

The correct response is Option B.

In this circumstance, the surgeon should wash the graft with sterile triple antibiotic solution (0.1% gentamicin, 0.1% clindamycin, 0.05% polymyxin) and consider reusing it. This graft is not easily re-harvestable, and therefore reusing it is a reasonable alternative. Triple antibiotic solution is readily available and nontoxic to the bone graft, unlike iodine and chlorhexidine. Studies have shown that iodine does not have the antimicrobial effectiveness of other antimicrobials. Studies also show that while chlorhexidine is most effective in eradicating the microorganisms from the graft, its detrimental effect on corticocancellous bone prevents its use in this clinical scenario. If the dropped graft were skin, cartilage, or composite tissue, chlorhexidine would be the ideal antimicrobial.

Discarding the graft is not the ideal alternative given the extremely low likelihood of infection if reused after appropriate decontamination. In this clinical scenario, alloplastic materials are contraindicated due to the patient’s age. One should change the wound classification from 1 to 3, not 4. Steam sterilization is not used for such grafts in such an acute setting and this technique would potentially harm the graft.

2017

How well did you know this?
1
Not at all
2
3
4
5
Perfectly
14
Q

A 16-month-old male infant is brought to the physician because of congenital anomalies of both feet. The dorsal and plantar aspects are depicted in the photographs shown. Which of the following is the most likely diagnosis?

A) Apert syndrome
B) Crouzon syndrome
C) Jackson-Weiss syndrome
D) Pfeiffer syndrome
E) Saethre-Chotzen syndrome

A

The correct response is Option A.

Syndromic craniosynostosis often presents with findings in the hands and feet, sometimes referred to as acrocephalosyndactyly.

Apert syndrome is unique for having bilateral symmetric complex syndactylies involving nearly all digits, of both the hands and feet.

Crouzon syndrome usually has normal hands and feet.

Jackson-Weiss can have foot anomalies, such as short metatarsals.

Pfeiffer syndrome usually has broad thumbs and toes.

Saethre-Chotzen syndrome may have incomplete single syndactylies, but otherwise does not typically have extremity findings.

2017

How well did you know this?
1
Not at all
2
3
4
5
Perfectly
15
Q

A 2-year-old boy is referred for evaluation of an abnormal frontal prominence that his parents report has become more noticeable during the past year. A photograph is shown. He is otherwise healthy and is meeting all developmental milestones. A CT scan obtained at the request of his pediatrician shows closure of the metopic suture with ectocortical thickening; no other abnormalities are noted. Which of the following is the most appropriate next step in management?

A) Endoscopic suturectomy and postoperative helmet therapy
B) Fronto-orbital advancement
C) Spring-mediated frontal distraction
D) Total calvarial remodeling
E) Observation

A

The correct response is Option E.

This patient has a metopic ridge and would not be correctly classified as having the phenotype associated with pathologic metopic closure—trigonocephaly. Consequently, observation is the only correct answer. Thickening of the metopic suture is a normal variant and should not be interpreted as abnormal unless it is accompanied by frontal narrowing and retrusion of the superior-lateral orbital rims. These findings are not present in this patient.

The metopic suture closes normally within the first year of life, so the presence of a fused metopic suture on CT scan in a child this age is not necessarily abnormal. The degree of frontal narrowing required to classify a patient as having pathologic cranial shape is a matter of much debate and is not settled. Some authors point to the importance of additional clinical (hypotelorism, biparietal widening) or radiographic (endocortical thickening or omega sign on CT) findings to secure the diagnosis of metopic craniosynostosis.

The other responses are surgical interventions and should be invoked only in the context of pathologic metopic closure and resultant trigonocephaly. Endoscopic suturectomy and postoperative helmet therapy, spring-mediated frontal distraction, and fronto-orbital advancement are all viable options to correct the frontal narrowing associated with trigonocephaly. Total calvarial remodeling is rarely if ever required to correct trigonocephaly.

2017

How well did you know this?
1
Not at all
2
3
4
5
Perfectly
16
Q

A newborn male with a Tessier No. 3 orofacial cleft is evaluated in the NICU. Which of the following locations is the most common pathway of this cleft through the alveolar ridge?

A) Between the canine and premolar
B) Between the central and lateral incisors
C) Between the central incisors
D) Between the lateral incisor and canine
E) Between the premolar and first molar

A

The correct response is Option D.

Tessier No. 3 orofacial clefts are the most common type. When they affect the alveolus, they typically traverse between the lateral incisor and the canine and extend into the floor of the nose and through the nasolacrimal system and orbital floor, involving the medial canthal region.

The other options are all incorrect, because they are not typically the route seen in the Tessier No. 3 orofacial cleft.

2017

How well did you know this?
1
Not at all
2
3
4
5
Perfectly
17
Q

The congenital anomaly shown in the photograph is thought to be caused by which of the following?

A) Failure of fusion of the lateral and maxillary nasal processes
B) Failure of fusion of the maxillary prominence with the medial nasal prominence
C) Failure of fusion of the medial nasal prominence and the lateral nasal prominence
D) Failure of fusion of the medial nasal prominences
E) Failure of the oronasal membrane to rupture

A

The correct response is Option A.

Proboscis lateralis, which is illustrated in the photograph, is thought to be a failure of fusion between the lateral and maxillary nasal processes. A cleft lip is the result of a failure of fusion of the maxillary prominence with the medial nasal prominence. A midline cleft or Tessier Zero cleft is a result of the failure of fusion of the medial nasal prominences. Choanal atresia is a result of a failure of the oronasal membrane to rupture. Finally, a mandibular cleft or Tessier # 30 cleft is a result of the failure of fusion of the mandibular prominences.

2017

How well did you know this?
1
Not at all
2
3
4
5
Perfectly
18
Q

A 12-year-old boy is evaluated for a vertical furrow near the midline of his face from the hairline to the eyebrows. Each of the listed clinical findings can be seen in hemifacial atrophy EXCEPT:

A) Atrophy of the tongue
B) Change in facial sensation
C) Malar hypoplasia
D) Malocclusion
E) Mandible hypoplasia

A

The correct response is Option B.

The relationship between morphea en coup de sabre and Parry-Romberg syndrome is unclear but there is some overlap. Morphea is characterized by the vertical furrow, atrophy of the tongue and upper lip, absent or flattened zygoma, orbital rim, and a hypoplastic maxilla and mandible on the affected side. A lateral open bite may be seen due to the maxillary and mandibular hypoplasia. Sensation, function of muscles of facial expression, and mastication are normal.

2017

How well did you know this?
1
Not at all
2
3
4
5
Perfectly
19
Q

A 20-year-old woman comes to the office to discuss reconstruction for hemifacial atrophy. A photograph is shown. The atrophy began when she was 13 years old and stabilized by 16 years of age. Workup suggests mixed connective tissue etiology that falls within the lupus spectrum. Which of the following is the most appropriate intervention to correct this patient’s deformity?

A) Buried anterolateral thigh free flap
B) Custom silicone prosthesis
C) Gracilis free flap
D) Injectable poly-L-lactic acid
E) Pedicled pectoralis major flap

A

The correct response is Option A.

Facial contour deformities can result from a variety of causes, can be unilateral or bilateral, and may range in severity. The most common cause is Romberg disease, progressive hemifacial atrophy. The cause is unknown but may be a localized form of scleroderma. Systemic lupus erythematosus and scleroderma are the most common connective tissue disorders resulting in soft-tissue atrophy with facial involvement.

Systemic lupus erythematosus is an inflammatory connective tissue disorder of unknown cause that predominantly affects women (90%). The clinical course can be variable and involve almost all organ systems. Cutaneous manifestations include soft-tissue atrophy, telangiectasia, and extremity ulcerations. Scleroderma refers to a spectrum of disorders characterized by diffuse fibrosis of the skin, blood vessels, synovia, and vital organs, such as the kidneys. Women are affected four times more frequently than men and the skin is involved in 95% of cases. The pathogenesis is unclear but may include vascular abnormalities (vasculitis), excess collagen deposition, and excess proteoglycan production in the involved tissues.

Surgical options for correction of hemifacial atrophy include alloplastic implants, free dermis/fat grafts, fat injections, and fasciocutaneous free tissue flaps. For mild defects, a custom alloplastic implant composed of silicone or porous polyethylene may be acceptable, but risks such as skin breakdown with implant exposure, scar contracture, and poor long-term patient satisfaction make an alloplastic option less desirable. In the patient described, the large nature of the defect and thin nature of her skin make an alloplastic option a poor choice.

Fat grafting using the techniques described by Coleman is a viable option for smaller defects. The large defect and need for multiple cycles of fat grafting make this a less desirable option in this patient. Various free flaps including omentum, parascapular, groin, and superficial inferior epigastric artery flaps have been described for correction of hemifacial atrophy. In this case, the anterolateral thigh flap is the most appropriate choice because it allows for harvest of a large amount of tissue, including skin, fat, fascia, and muscle. A postoperative photograph is shown.

2016

How well did you know this?
1
Not at all
2
3
4
5
Perfectly
20
Q

A 3-month-old male infant is evaluated because of bilateral preauricular pits and small ears. Physical examination is otherwise unremarkable. The patient failed the newborn hearing test; the parents have not repeated the test. They are not very concerned, because the father is also hearing impaired. Which of the following is the most appropriate next step in management?

A) Echocardiography
B) MRI of the brain
C) Renal ultrasound
D) Repeat hearing test
E) Surgical treatment of the preauricular pits

A

The correct response is Option C.

This child most likely has branchio-oto-renal (BOR) syndrome, an autosomal dominant syndrome. Two genetic mutations (EYA1 and SIX1) have been identified. The clinical findings include auricular malformations, preauricular skin pits, hearing loss, branchial fistulae, external auditory canal stenosis, and renal anomalies. Renal ultrasound should be performed to rule out renal anomalies. Family history of hearing loss should also raise suspicion.

Ear anomalies can also be associated with the oculo-auriculo-vertebral spectrum (OAV), within which hemifacial microsomia falls. OAV spectrum may have associated mandibular, facial nerve, and ocular findings (e.g., epibulbar dermoid, coloboma). Other possible associations include congenital heart defects, such as ventricular septal defect or tetralogy of Fallot. This child does not have other craniofacial findings of hemifacial microsomia, thus cardiac echocardiography is not indicated.

Internal cerebral arterial anomalies may be associated with PHACE syndrome (P, posterior fossa; H, hemangioma; A, arterial anomalies; C, cardiac defects; E, eye anomalies). Investigation of cerebral circulation by MRI of the brain would be prompted by a facial hemangioma, which is absent in this child.

While resection of the preauricular pits may be offered, it does not aid in ruling out other findings and there is no urgency to perform surgical resections in this child at 3 months of age.

Repeating a hearing test will not confirm or rule out OAV or BOR syndrome.

2016

How well did you know this?
1
Not at all
2
3
4
5
Perfectly
21
Q

A 12-hour-old male newborn is evaluated in the neonatal intensive care unit. A diagnosis of CHARGE syndrome is suspected. Review of the medical record and physical examination show congenital heart defect, eyelid colobomas, microtia, hypogonadism, and growth retardation. Which of the following additional conditions is/are most likely in this patient?

A) Craniosynostosis
B) Intrahepatic hemangiomas
C) Lower lip pits
D) Orofacial cleft
E) Syndactyly

A

The correct response is Option D.

Cleft lip and/or palate is found in up to 20% of patients with CHARGE (coloboma of the eye, heart defects, atresia of the nasal choanae, retardation of growth and/or development, genital and/or urinary abnormalities, and ear abnormalities and deafness) syndrome. It is the second most common syndrome associated with cleft lip and palate, following van der Woude syndrome. CHARGE syndrome is believed to be caused by a microdeletion or mutation in the CHD7 (chromodomain helicase DNA-binding protein 7) gene. Over the years, there have been other conditions found to be associated with this syndrome, including limb anomalies, renal anomalies, omphalocele, cranial nerve anomalies, and tracheoesophageal fistula.

Syndactyly is not typically associated with CHARGE syndrome.

Intrahepatic hemangiomas are not typically associated with CHARGE syndrome.

Craniosynostosis is not typically associated with CHARGE syndrome.

Lip pits are commonly found in patients with van der Woude syndrome.

2016

How well did you know this?
1
Not at all
2
3
4
5
Perfectly
22
Q

A 6-year-old girl with 22q11.2 deletion syndrome is brought to the office by her parents. The parents report trouble understanding the child when she speaks. An incompetent velopharyngeal port is suspected. Which of the following is the most likely indication of this malfunction?

A) Consonant omission
B) Glottal stop
C) Hyponasality
D) Nasal air emission
E) Sibilant lateralization

A

The correct response is Option D.

Nasal air emission, along with hypernasality and facial grimacing, are all findings that are commonly observed in patients with an incompetent velopharyngeal port or velopharyngeal dysfunction (VPD). Consonant omissions and glottal stops are two common pathologic compensatory speech patterns that such patients exhibit but are not directly caused by VPD. Instead, they are maladaptive compensatory speech patterns often present in patients with VPD. Although some patients with VPD can have hyponasality or sibilant lateralization, these are unrelated to VPD or an attempt to compensate for VPD.

2016

How well did you know this?
1
Not at all
2
3
4
5
Perfectly
23
Q

A 3-month-old female infant is brought to the office for evaluation of head shape. Examination shows brachycephaly and abnormal orbits. Genetic testing is negative for fibroblast growth factor receptor (FGFR) mutation. Which of the following is the most likely diagnosis?

A) Antley-Bixler syndrome
B) Apert syndrome
C) Crouzon syndrome
D) Pfeiffer syndrome
E) Saethre-Chotzen syndrome

A

The correct response is Option E.

Only Saethre-Chotzen syndrome is not associated with a fibroblast growth factor receptor (FGFR) mutation, but it is instead associated with a mutation of the TWIST gene. It is characterized by brachycephaly with maxillary hypoplasia, a prominent ear crus, and syndactyly. Pfeiffer syndrome (characterized by brachycephaly, mild syndactyly, and broad toes and thumbs) is associated with mutations of both FGFR1 and FGFR2. Apert syndrome (characterized by brachycephaly, syndactyly, and cognitive delay) is autosomal dominant and associated with mutations of FGFR2. Crouzon syndrome is also autosomal dominant and characterized by shallow orbits, craniosynostosis, and maxillary hypoplasia. Antley-Bixler syndrome has four phenotypes, some of which are associated with FGFR mutations and characterized by craniosynostosis, choanal atresia, and radiohumeral synostosis.

2016

How well did you know this?
1
Not at all
2
3
4
5
Perfectly
24
Q

A 3-month-old male infant is brought to the office by his parents for evaluation of skull asymmetry that has worsened since birth. Birth history includes prolonged labor that required cesarean delivery. Physical examination shows flattening of the right posterior occiput with ipsilateral forehead bossing. From a superior view, the right ear is 1 cm more anterior than the left ear, and the anterior fontanelle is open without any bulging. The child’s head tilts to the right and has decreased range of motion when looking to the left. The left side of his neck feels tighter and more rigid compared with the right side. Which of the following is the most appropriate initial management of this child?

A) Cerebral palsy evaluation
B) Occupational therapy of the neck
C) Posterior cranial vault expansion
D) Passive molding helmet therapy
E) Sternocleidomastoid muscle release

A

The correct response is Option B.

This child demonstrates the classic presentation of deformation plagiocephaly with his posterior occiput flattening and compensatory ipsilateral forehead bossing with anterior shifting of his ear on the same side. There are many reasons for deformational plagiocephaly, especially with the current practice of “back to sleep.” Treatment of this focuses on removing the pressure on the affected side. His head tilt and decreased motion are consistent with torticollis. Initial treatment is stretching and occupational therapy to restore usage and balance of his neck muscles. Although helmet therapy can help alleviate pressure on this side, it is not addressing the issue. Further studies have demonstrated that deformational plagiocephaly can be treated with positional changes and behavior modification up until 7 to 8 months of age without difference in head asymmetry compared with those children who began helmet therapy at a younger age. There is no difference in children who fail positional changes compared with those who initiate helmet therapy immediately. At this child’s age of 3 months and with obvious torticollis, the most appropriate initial therapy should be focused on resolving his torticollis and giving him a trial of non-helmet therapy.

2016

How well did you know this?
1
Not at all
2
3
4
5
Perfectly
25
Q

A 3-year-old girl with craniofrontonasal dysplasia presents to the craniofacial team because of moderate exorbitism, severe central sleep apnea, and severe hypertelorism (interocular distance of 40 mm). Which of the following is the most appropriate management plan for this patient?

A) Continuous positive airway pressure until 6 or 8 years of age
B) Facial bipartition surgery within the next year
C) Immediate tracheostomy and assisted ventilation
D) Monobloc advancement surgery within the next year
E) Orbital box osteotomy surgery within the next year

A

The correct response is Option C.

Tracheostomy and assisted ventilation is the most appropriate management. The patient described demonstrates a significant number of central and mixed apneas, indicating that the initiation of breathing by central drive is diminished and that tracheostomy with assisted ventilation set at a base ventilation rate, such as synchronized intermittent mandatory ventilation (SIMV), would be optimal to ensure adequate minute ventilation.

Continuous positive airway pressure (CPAP) until 6 to 8 years of age and then a facial advancement is incorrect. CPAP will assist with obstructive apnea, but will not treat central apnea because no baseline ventilation rate is set using CPAP mode.

Hypertelorism surgery within the next year is incorrect because hypertelorism surgery will not treat sleep apnea without facial advancement. Only the wide orbit position would be treated with an orbital box osteotomy or facial bipartition without advancement.

Facial advancement surgery within the next year is incorrect because advancement will correct the obstructive apnea and a portion of the mixed apneas, but will not address the central sleep apnea. Observation with tracheotomy and assisted ventilation will be more prudent until the central apnea rate improves, usually after time with maturation. The patient described will still have persistent central apnea and is at higher risk for relapse of the face due to the age of under 6 years.

2016

How well did you know this?
1
Not at all
2
3
4
5
Perfectly
26
Q

Which of the following arteries is most likely to be injured in a patient undergoing Le Fort I osteotomy?

A) Descending palatine
B) External carotid
C) Greater palatine
D) Internal carotid
E) Internal maxillary

A

The correct response is Option A.

Descending palatine artery is the correct response since it is at greatest risk after pterygoid osteotomy and down fracture. It is easily visible after down fracture and mobilization of the Le Fort I segment. It is typically injured during osteotomy prior to its division into the greater and lesser palatine arteries.

External carotid artery is incorrect because it is within the neck. Its branches ascend in the face and end in the sphenopalatine artery as it enters the pterygoid foramen.

Internal maxillary artery is incorrect because it is the larger artery prior to division into multiple smaller branches including the middle meningeal, sphenopalatine, and descending palatine arteries.

Greater palatine artery is incorrect because the greater and lesser palatine arteries are below the level of the pterygoid osteotomy. The descending palatine is more proximal and is at greater risk for injury.

2016

How well did you know this?
1
Not at all
2
3
4
5
Perfectly
27
Q

A 6-month-old male infant is evaluated for facial asymmetry noted at birth. Physical examination shows symmetry of the upper one-third of the face, but the right cheek is less prominent than the left. The occlusal plane is canted upward to the right, and the chin point is deviated to the right. In addition, the right ear is small and constricted, and an adjacent preauricular soft-tissue appendage is noted. He has a palsy of the right marginal mandibular nerve. He is otherwise healthy. The parents should be informed that the asymmetry was most likely caused by which of the following?

A) Autosomal dominant mutation
B) Disruption of normal developmental sequence
c) Maternal smoking
D) Omega oil ingestion during pregnancy
E) Uterine deformation

A

The correct response is Option B.

This patient has hemifacial microsomia (also referred to as craniofacial microsomia, Goldenhar syndrome, or oculoauriculovertebral spectrum), the second most common congenital facial malformation after cleft lip/palate. It is estimated to occur in 1:5600 to 1:20,000 live births. Unilateral (or less commonly, bilateral) facial hypoplasia is the primary manifestation, and it can involve ocular structures (orbital size, position, globe development, coloboma, epibulbar dermoids), mandibular structures, auricular structures (preauricular tags and sinuses, microtia or anotia, hearing loss), cranial nerve palsies (cranial facial nerve [VII] is most common); and soft-tissue underdevelopment (including facial clefts). In addition, patients can have cardiac, musculoskeletal, central nervous system, and renal manifestations.

The etiology of hemifacial microsomia (HFM) appears to be heterogeneous. Early investigators using animal models of the condition believed that this was the result of an isolated vascular event in the stapedial artery. While this mechanism cannot be completely dismissed and may have a role in some cases, the prevalence of extracranial findings strongly suggests a more sublime pathogenesis: faulty neural crest cell (NCC) migration. Both genetic and environmental factors have been demonstrated to alter or impair NCC activity. Recent studies have implicated genetic factors in the development of this disorder. Nevertheless, the majority of patients with HFM have no family history and less than 2% of cases arise from autosomal dominant inheritance. Certain vasoactive and nonsteroidal anti-inflammatory drug (NSAID) medications can increase the risk of HFM; maternal ingestion of omega oils has not been associated. Maternal smoking can increase the risk of having a child with HFM, but the association is weak and this is not causative in the vast majority of patients. Uterine deformation can lead to facial asymmetry; however, this typically improves with growth and does not lead to the ear and jaw anomalies in the patient described.

2016

How well did you know this?
1
Not at all
2
3
4
5
Perfectly
28
Q

A 30-year-old primigravid woman is referred to the office because ultrasonography showed polyhydramnios at 22 weeks’ gestation. Additionally, the fetus was observed to have macrosomia, omphalocele, macroglossia, and microtia. There is no evidence of nephromegaly or hepatomegaly. A groove between the lobule and antitragus is noted. Chromosomal analysis indicates rearrangements on chromosome 11p15. Which of the following is the most likely diagnosis?

A) Beckwith-Wiedemann syndrome
B) Binder syndrome
C) Diabetic fetopathy
D) Down syndrome
E) van der Woude syndrome

A

The correct response is Option A.

This is a rare overgrowth disorder involving genetic defect of chromosome 11, commonly characterized by the presence of macrosomia, omphalocele, and macroglossia. These symptoms typically present after 22 weeks’ gestation. Additionally, infants with Beckwith-Wiedemann syndrome often display auricular abnormalities, including characteristic ear folds.

While a common symptom of diabetic fetopathy is macrosomia, a diagnosis of diabetic fetopathy would be incorrect. Common symptoms of diabetic fetopathy include congenital anomalies of the internal organs, such as nephromegaly or hepatomegaly; no such symptoms are detected in this fetus.

The absence of cleft lip and/or cleft palate nephromegaly also precludes the diagnosis of van der Woude syndrome. Additionally, the genetic abnormality responsible for van der Woude syndrome involves chromosome 1, not chromosome 11. Likewise, trisomy 21 (Down syndrome) is not the correct diagnosis, as the chromosomal abnormality responsible for this disease involves chromosome 21, not chromosome 11.

Finally, Binder syndrome is a congenital facial malformation most commonly characterized by a flat nose and retrusion of the midface, neither of which are observed in this fetus. Therefore, Beckwith-Wiedemann syndrome is the correct answer.

2016

How well did you know this?
1
Not at all
2
3
4
5
Perfectly
29
Q

A 10-month-old male infant has undergone a cranial vault remodeling for sagittal synostosis. The child has no other comorbidities. On the second postoperative day, the patient’s serum sodium concentration decreases from 140 to 129 mEq/L. Laboratory studies show a suppressed plasma aldosterone concentration. The suspected diagnosis is cerebral salt wasting (CSW). An increase in which of the following is most likely to support this diagnosis?

A) Plasma antidiuretic hormone concentration
B) Plasma sodium concentration
C) Plasma volume
D) Serum uric acid concentration
E) Urine output

A

The correct response is Option E.

Cerebral salt wasting (CSW) is a condition that combines renal sodium and fluid loss after intracranial injury with an overall negative sodium balance and volume contraction. It responds to fluid and salt replacement. It contrasts with the syndrome of inappropriate antidiuretic hormone (SIADH), which is part of the differential diagnosis in hyponatremia as follows:

2016

How well did you know this?
1
Not at all
2
3
4
5
Perfectly
30
Q

A 2-day-old male newborn is evaluated for the skull findings shown in the CT scan. Which of the following best describes the anomaly in this patient?

A) Brachycephaly
B) Kleeblattschädel deformity
C) Posterior plagiocephaly
D) Scaphocephaly
E) Trigonocephaly

A

The correct response is Option B.

This CT scan shows craniosynostosis of multiple sutures including the coronal, lambdoid, and a portion of the sagittal suture, which is characteristic of a Kleeblattschädel deformity. Brachycephaly is characterized by bicoronal craniosynostosis alone and is most commonly seen in syndromic craniosynostosis. Scaphocephaly is isolated involvement of the sagittal suture and is the most common type of craniosynostosis. Metopic synostosis is the cause of trigoncephaly and this suture is open in the CT scan. Lambdoid, which produces posterior plagiocephaly synostosis, is seen in the CT scan but not in isolation and is the least common of the single suture synostoses.

How well did you know this?
1
Not at all
2
3
4
5
Perfectly
31
Q

A 10-year-old boy with a history of fronto-orbital advancement for craniosynostosis as an infant is brought to the office for evaluation. He would like to play football, but his parents are concerned about a persistent calvarial defect. Physical examination shows a 4 × 4-cm area of the right forehead that has no bony coverage; dural pulsations are observed. Which of the following is the most appropriate management?

A) Fabrication of a custom implant
B) Reconstruction with methylmethacrylate
C) Split calvarial bone grafting
D) Split rib grafting
E) Observation and delayed treatment until after completion of growth

A

The correct response is Option C.

This is a growing child with a large calvarial defect in a problematic location. Therefore, the defect needs to be addressed sooner rather than later, due to the potential risk for trauma and the child’s desire to be active. The gold standard for reconstruction of defects of this size and location is split calvarial bone grafting. Rib grafting would work, but would necessitate another donor site and contouring issues. A custom implant or methylmethacrylate would fix the problem in the short term, but would not grow with the child, necessitating additional surgery in the future.

2015

How well did you know this?
1
Not at all
2
3
4
5
Perfectly
32
Q

A 6-month-old girl is brought to the office for evaluation of an isolated cleft of the soft palate. A photograph is shown. Which of the following genes is most likely defective in this patient?

A) FGFR2

B) IRF6

C) MSX1

D) SHH

E) TGF-beta

A

The correct response is Option B.

The patient has Van der Woude syndrome, an autosomal dominant condition affecting 1:35,000 to 1:100,000 persons. The pathognomonic features include lower lip pits, as seen in the photograph, and clefting of the palate, lip, or both. This syndrome is the most common single-gene cause of cleft lip and palate. IRF6 codes for a transcription factor that is involved in the early development. The mutated copy of the gene decreases the amount of active protein and results in the defects associated with this syndrome and popliteal pterygium syndrome. The remaining genes and their products have been implicated in craniofacial morphogenesis and etiopathogenesis of cleft lip/palate. Nevertheless, none are associated with lip pits or are an autosomal dominant cause of facial clefting.

2015

How well did you know this?
1
Not at all
2
3
4
5
Perfectly
33
Q

According to the Tessier cleft classification types, at which of the following areas is the most common facial cleft centered?

A) Frontonasal suture
B) Frontozygomatic suture
C) Nasomaxillary buttress
D) Superior orbit
E) Zygomaticotemporal suture

A

The correct response is Option E.

Patients with rarer facial clefts are classified according to the system proposed by Tessier in 1973. Each cleft is classified by the soft-tissue findings and the bony defects as they relate to the orbit. Nos. 0 through 7 occur in the lower half of the face as delineated by the orbital commissure, and Nos. 9 through 14 occur in the upper half of the face. The most common facial cleft is the No. 7 cleft centered in the region of the zygomaticotemporal suture. It includes variable expression of macrostomia and hypoplasia of the zygoma.

How well did you know this?
1
Not at all
2
3
4
5
Perfectly
34
Q

Which of the following congenital clefts involves the lower eyelid?

A) 0
B) 1
C) 5
D) 8
E) 9

A

The correct response is Option C.

(Please note that this pictorial appears in color in the online examination)

Craniofacial clefts are highly variable and can range from mild forme fruste to severe with involvement of all the layers of the soft tissue and skeletal structures. Tessier described a classification system still in use today based on the bony and soft-tissue landmarks involved in the clefts. Corrective surgery is required in stages, with the timing based on the level of functional problems, including ocular exposure, and airway problems early on. The clefts can be broadly grouped into midline and paramedian clefts (numbers 0 to 14, 1 to 13, 2 to 12), oro-nasal-ocular clefts (numbers 3 to 11, 4 to 10, 5 to 9), and lateral clefts (numbers 6, 7, and 8). In this system, the numbers 0 to 6 refer to clefts below the orbital and numbers 8 to 14 are above the orbit, with 7 being truly lateral.

How well did you know this?
1
Not at all
2
3
4
5
Perfectly
35
Q

A 4-month-old male infant is brought to the office for evaluation of an abnormal head shape that has been present since birth. The patient is developmentally appropriate and has no other abnormalites. A photograph is shown. At which of the following ages does the involved suture normally fuse?

A) 2 weeks
B) 8 months
C) 3 years
D) 10 years
E) 22 years

A

The correct response is Option B.

The patient in the photograph has metopic craniosynostosis. The primary clinical features of trigonocephaly are a palpable metopic ridge, bossing with thickened bone at the glabella, bifrontal narrowing, recession of the superior orbital rims, temporal narrowing, and hypotelorism. Trigonocephaly is also known as a keel-shaped deformity.

The metopic suture is the first cranial suture to fuse and this usually occurs at approximately 8 months of age. The other cranial sutures generally fuse in adulthood.

Treatment depends on the age of presentation and extent of deformity. Options for management are varied and range from no intervention for minimal deformity, burring of the metopic ridge only, endoscopic synostectomy with helmet therapy, and bilateral fronto-orbital advancement.

2015

How well did you know this?
1
Not at all
2
3
4
5
Perfectly
36
Q

A 23-year-old man comes to the office because of a progressive 15-year history of severe unilateral volume loss in the face. The patient says he is unhappy with the changes in his facial appearance but has not noted any changes recently. On examination, volume loss is significant and appears to be limited to the soft tissue. No bony deficit is noted. Which of the following is the most appropriate management?

A) Alloplastic bony augmentation of the maxilla
B) Autologous soft-tissue augmentation with a cross-facial nerve graft followed by microvascular gracilis muscle transfer
C) Autologous soft-tissue augmentation with a microvascular parascapular flap
D) Autologous soft-tissue augmentation with a pedicled temporalis muscle flap
E) Hyaluronic acid augmentation of the soft tissues

A

The correct response is Option C.

It is most likely that this patient has progressive, but now stable, hemifacial atrophy, or Parry-Romberg syndrome. The course is characterized by progressive unilateral atrophy of the soft tissues of the face. The underlying skeleton may be affected in severe forms of the disease. Surgery is indicated when the changes stabilize. The recommended treatment is free tissue transfer, preferably a microvascular parascapular flap, followed by a secondary autologous fat grafting for refinement. Augmentation of the maxilla is not indicated in the absence of bony atrophy. Staged cross-facial nerve grafting followed by microvascular gracilis transfer is indicated for absence of facial nerve function. A pedicled temporalis muscle flap would likely not provide sufficient bulk where needed and potentially leave a hollow at the donor site. Alloplastic augmentation of the soft tissues with hyaluronic acid is only approved for volume loss due to human immunodeficiency virus infection.

2015

How well did you know this?
1
Not at all
2
3
4
5
Perfectly
37
Q

A 9-year-old boy is brought to the office because of a long-standing history of pulsatile exophthalmos and proptosis of the right side. CT scan shows absence of the greater sphenoid wing. Which of the following is the most likely diagnosis?

A) Hemifacial microstomia
B) Neurofibromatosis
C) Tessier No. 5 cleft
D) Treacher Collins syndrome
E) Unilateral coronal synostosis

A

The correct response is Option B.

Although each of the disorders or syndromes listed is associated with a periocular soft-tissue or orbital abnormality, only neurofibromatosis is associated with agenesis of the sphenoid wing.

While plastic surgeons primarily manage the soft-tissue manifestations of neurofibromatosis, the disorder is associated with skeletal abnormalities including sphenoid wing aplasia, macrocephaly, scoliosis, and tibial bowing. Sphenoid wing aplasia causes brain herniation through the middle cranial fossa into the orbit with pulsatile exophthalmos, vertical dystopia, and an enlarged orbital volume. Surgical correction involves bone grafts or titanium mesh to reconstruct the sphenoid wing with reduction of the temporal lobe into the middle cranial fossa.

Unilateral coronal synostosis is characterized by foreshortening of the orbital roof on the affected side. Synostosis of the coronal suture causes superior displacement of the lesser wing of the sphenoid with the radiographic appearance of the harlequin deformity. Patients may have strabismus occurring secondary to relative paresis of the superior oblique muscle. The child may tilt the head to compensate. Treacher Collins syndrome is associated with lower eyelid colobomas, zygomatic hypoplasia with hypoplastic or absent zygomas and inferolateral orbital rim clefting, absence of eyelashes, and bilateral mandibular hypoplasia. Hemifacial microstomia is a variant of craniofacial microsomia, distinguished by the presence of concomitant ocular abnormalities, including epibulbar dermoids. The Tessier No. 5 cleft begins behind the canine and extends through the maxillary sinus to the orbital floor. Colobomas of the lateral lower eyelids and clefting of the upper lip medial to the oral commissure are associated.

2015

38
Q

A 12-year-old girl is evaluated for left-sided progressive hemifacial atrophy. Onset of symptoms started at 10 years of age. She has a history of migraines and experienced a seizure 1 year ago. Physical examination shows left-sided facial atrophy including cheek depression, enophthalmos, and dry skin with hyperpigmentation and alopecia. Maxillofacial CT scan shows associated skeletal changes. Which of the following clinical features is most likely associated with the early onset of the disease process?

A) Enophthalmos
B) Facial paralysis
C) Hyperpigmentation
D) Seizure disorder
E) Skeletal atrophy

A

The correct response is Option E.

Progressive hemifacial atrophy, also known as Parry-Romberg syndrome, is a rare disorder characterized by a self-limiting, slow, progressive, unilateral facial atrophy affecting the skin, subcutaneous tissue, muscle, and osteocartilaginous adjacent structures. It involves one or more trigeminal dermatomes with symptoms usually appearing in the first or second decade of life. The progression of atrophy usually lasts a variable 2 to 10 years. The etiology and pathogenesis of this disease are uncertain.

The severity of soft-tissue involvement appears to be independent of age of onset, facial distribution, or extent of the disease process; however, age of onset has been shown to correlate with the degree of bony hypoplasia. Onset of the disease at an earlier age during skeletal immaturity has a higher likelihood of skeletal involvement.

Clinical features of hemifacial atrophy include enophthalmos, cheek depression, deviation of the mouth and nose towards the affected side, ipsilateral show of teeth, and tongue atrophy. In addition, extracutaneous involvement has been described, including hemiatrophy of the contralateral or ipsilateral arm, trunk or leg, as well as dental, ocular, and neurologic abnormalities such as enophthalmos, headaches, facial pain, and seizures. An age correlation with these features has not been demonstrated. Facial paresthesia has been described but motor function remains intact.

2015

39
Q

A 6-month-old girl is evaluated because of a “flat head.” Physical examination shows right posterior flatness of the cranium with a transcranial difference of 7 mm (minimal to moderate), and mastoid bossing on the ipsilateral side. The right ear and left forehead are anteriorly displaced. The parents report that the child prefers to turn her head to the right. The patient is otherwise healthy and reaching developmental milestones. Which of the following is the most appropriate next step in management?

A) Cranial orthotic molding helmet
B) Crib positioning and physical therapy only
C) CT scan of the head
D) X-ray studies of the skull
E) Reassurance

A

The correct response is Option C.

The clinical findings are suggestive of right unilateral lambdoidal synostosis (synostotic plagiocephaly). If craniosynostosis is suspected, the child may need surgical correction of the defect. Therefore, it is incorrect to pursue reassurance and conservative therapy without confirming the presence of a prematurely fused suture.

Lambdoidal synostosis is the least common premature fusion of a suture (2 to 3% of all synostosis cases). The physical findings have some similarities and distinct differences from the frequently seen positional deformational plagiocephaly. Positional plagiocephaly can have ipsilateral forehead protuberance and anterior displacement of the ipsilateral ear. Lambdoid synostosis, on the other hand, is associated with ipsilateral mastoid bossing and protuberance of the contralateral forehead. The position of the ear, once another landmark to help distinguish between positional and synostotic plagiocephaly, with lambdoid synostosis associated with posterior displacement of the ipsilateral ear, has been questioned in recent years. A literature review showed that the ipsilateral ear has been reported to be nondisplaced, anteriorly displaced, or posteriorly displaced. Therefore, the significance of the position of the ear is unclear at this time.

There is no need for plain film series before head CT scan. Imaging studies are performed to confirm synostosis before surgical intervention.

2015

40
Q

An 8-hour-old male newborn is examined in the neonatal intensive care unit for epibulbar dermoids. X-ray studies show fused cervical vertebrae. Which of the following additional features is most consistent with this diagnosis?

A) Cleft palate
B) Lower lip pits
C) Microtia
D) Syndactyly
E) Telecanthus

A

The correct response is Option C.

Oculoauricular dysplasia (Goldenhar syndrome) may have many different possible manifestations, but is typically characterized by ear anomalies, epibulbar dermoids, facial and mandibular hypoplasia, and vertebral anomalies.

Cleft palate, although it may be encountered in patients with Goldenhar syndrome, is not generally regarded as a classic finding and is less common than microtia in affected patients.

Lower lip pits are a finding suggestive of van der Woude syndrome, the most common orofacial clefting syndrome, rather than Goldenhar syndrome.

Syndactyly is not generally regarded as a component of Goldenhar syndrome.

Telecanthus is not a classic finding in patients with Goldenhar syndrome.

2014

41
Q

A 4-week-old male newborn is evaluated for complete left unilateral cleft of the lip and palate. Genetics workup shows IRF6 gene mutation. Which of the following additional physical findings is most likely in this patient?

A) Agenesis of the corpus callosum
B) Glossoptosis
C) Hypodontia
D) Medialized internal carotid arteries
E) Preauricular accessory tag

A

The correct response is Option C.

The mutation for van der Woude syndrome has been mapped to the interferon regulatory factor 6 (IRF6) gene in chromosome 1. The inheritance is autosomal dominant with variable penetrance. Other associated findings include hypodontia (as high as 86%), high arched palate, lip pits, syngnathia, and ankyloglossia. Agenesis of the corpus callosum and preauricular skin tags are not associated with van der Woude syndrome.

Glossoptosis is associated with Pierre Robin sequence. Medialized internal carotids can be seen in patients with velocardiofacial syndrome.

2014

42
Q

The association between craniofacial defects and cardiac malformations in patients with velocardiofacial syndrome results from a disruption in the cellular development of which of the following?

A) Cardiogenic mesoderm
B) Ectodermal placodes
C) Lateral plate mesoderm
D) Neural crest
E) Somites

A

The correct response is Option D.

Neural crest cells derive from the ectoderm layer of the developing embryo, specifically the neuroectoderm of the forebrain, midbrain, and hindbrain. The neural crest contributes significantly to the craniofacial region, and also to the conotruncal endocardial cushions that are responsible for dividing the outflow tract of the heart into separate pulmonary and aortic components. Therefore, defects in neural crest cell development will frequently result in malformations of both the craniofacial area and cardiac septum. Examples of this association are: Treacher Collins syndrome, Pierre Robin sequence, 22q11.2 deletion syndrome, and oculoauriculovertebral syndrome.

The cardiogenic mesoderm derives from splanchnic (visceral) mesoderm, and contributes the precursor cells that differentiate into the endocardium and myocardium. It does not contribute to the head and neck.

Ectodermal placodes are separate from the neuroectoderm and consist of areas of thickened ectoderm that guide neural crest cells in forming the cranial sensory ganglia.

Lateral plate mesoderm is a derivative of the mesoderm layer of the embryo and contributes to the laryngeal cartilages and associated connective tissue of the head and neck.

Paraxial mesoderm produces somites that form the skull, meninges, voluntary craniofacial musculature, and dermis and connective tissue of the dorsal aspect of the head.

2014

43
Q

A female newborn is evaluated after an uneventful delivery because of microtia of the left ear. The face appears otherwise symmetrical. Which of the following studies is most appropriate to obtain?

A) CT scan to rule out tethered cord
B) Echocardiography to rule out ventricular septal defect
C) Extremity x-ray studies to rule out limb-length discrepancies
D) Magnetic resonance angiography of the brain to rule out ipsilateral cerebral artery anomalies
E) Ultrasonography of the kidney to screen for structural anomalies

A

The correct response is Option E.

Syndromic ear anomalies are associated with an increased risk of renal anomalies in syndromes such as brachio-oto-renal syndrome, Townes-Brocks syndrome, etc.

Cardiac abnormalities are not associated with isolated microtia, but they are associated with extended spectrum hemifacial microsomia (oculoauriculovertebral dysplasia).

Magnetic resonance angiography of the brain to rule out internal cerebral artery anomalies is indicated in children suspected of PHACE syndrome (P, posterior fossa; H, hemangioma; A, arterial anomalies; C, cardiac defects; E, eye anomalies) when clinical symptoms include a large segmental hemangioma.

Tethered cord is not associated with ear anomalies. It is a concern with myelomeningoceles and spina bifida, and with Chiari malformations.

Limb-length discrepancies are not associated with microtia.

2014

44
Q

A 28-year-old man is evaluated because of the facial deformity shown in the photograph. Three years ago, he underwent resection of an infratemporal malignancy and intraoperative alloplastic reconstruction of bony defects. Postoperatively, he underwent extensive radiation therapy. Which of the following is the most appropriate method for restoring facial volume in this patient?

A) Custom-fabricated alloplastic implantation
B) Dermal fat grafting
C) Implantation of layered acellular dermis
D) Parascapular free flap reconstruction
E) Serial fat grafting

A

The correct response is Option D.

The patient described has marked loss of facial soft-tissue volume related to the initial tumor resection and the adverse effects of postoperative radiation treatment. The best method for restoring soft-tissue volume is a scapular free flap. This method of reconstruction has advantages over the others listed. The free scapular flap does not rely on the damaged and scarred soft-tissue envelope for vascular support and, thus, it will retain its volume. In contrast, fat grafting, dermal fat graft, and layered acellular dermis all undergo some resorption, especially in this poorly vascularized recipient site. The scapular flap is of sufficient thickness to correct even a volume defect of this size. Although the other soft-tissue reconstructive options can improve contour, the volume required in this patient cannot be achieved with these modalities alone. The use of an alloplastic reconstruction alone can improve mid-facial volume, but will not address the lower third deficit. In addition, there is a moderate risk of extrusion and/or infection with this technique alone.

2014

45
Q

An 8-month-old female infant is brought to the office by her parents. Physical examination shows a wedge-shaped skull with a keel formation on the forehead, close-set eyes, and hollowness of the temporal fossa on both sides of the head. Premature cranial suture ossification at which of the following sites is the most likely cause of this patient’s condition?

A) Bicoronal
B) Lambdoid
C) Metopic
D) Sagittal
E) Unicoronal

A

The correct response is Option C.

Craniosynostosis refers to the premature fusion of one or more cranial sutures that make up the cranial vault and cranial base. Once this fusion occurs prematurely, the growth of the skull is altered and the development of the head takes on a characteristic morphologic shape that is determined by the fusing suture.

Trigonocephaly is classically characterized by a typically wedge-shaped skull when viewed from above; it originates from a premature stenosis of the metopic suture followed by a bilateral growth restriction of the forehead. This results in bitemporal narrowing and hypotelorism.

Plagiocephaly or unilateral coronal synostosis is characterized by the flattening of the forehead and frontoparietal region ipsilateral to the fused suture. As a result of this fusion, a compensatory bulge occurs in the opposite frontoparietal skull. The temporal fossa on the side of the fusion is convex and the ear becomes anteriorly displaced. The petrous portion of the temporal bone that contains the glenoid fossa is also displaced forward and the articulation with the mandible is displaced forward as a result. The nasal radix is also deviated toward the fused side and the tip of the nose is turned to the opposite side.

Ridging of the sagittal suture forms a narrow biparietal skull. Scaphocephaly shows compensatory growth in the frontal region or frontal bossing and/or occipital coning. There is associated enlargement of the head circumference. Sagittal synostosis remains the most frequent of the nonsyndromic craniosynostosis.

Unilateral lambdoid synostosis has ridging of the lambdoid suture, ipsilateral parieto-occipital flattening, prominence of the mastoid air cells, posterior displacement of the ear on the side of the occipital flattening, and scoliosis of the base of the skull, resulting in curvature of the cervical spine.

2014

46
Q

A 4-month-old female infant is brought to the office for evaluation. A photograph is shown. Which of the following additional abnormalities are most likely associated with this patient’s condition?

A) Glossoptosis and cleft palate
B) Hydronephrosis of the kidneys and hearing loss
C) Lacrimal duct obstruction and coloboma of the eyelids
D) Posterior fossa abnormalities and stenotic cerebral arteries
E) Supernumerary teeth and duplicate maxilla

A

The correct response is Option E.

The patient described has bilateral macrostomia, also known as Tessier No. 7 cleft, the most common facial cleft in the Tessier classification system. This resulted from the failure of fusion between the maxillary and mandibular processes. Repair of the macrostomia can be undertaken in the first months of life. Duplicated maxilla has been reported in as high as 39% of patients with macrostomia. It is defined as having multiple supernumerary teeth and marked overlap of the maxillary arches. Other craniofacial findings such as mild mandibular/condylar anomalies and alveolar clefting have also been reported. The anatomy can be defined by three-dimensional CT scan and panoramic x-ray study (Panorex). Therefore, it is important to continue to observe these children with dental and orthodontic workups as they grow, even after the repair of the macrostomia.

Glossoptosis and cleft palate are associated with Pierre Robin sequence. Renal anomalies may be associated with congenital anomalies of the ears, such as in branchiootorenal syndrome.

Posterior fossa abnormalities and intracranial arterial anomalies are associated with PHACE syndrome. (P, posterior fossa; H, hemangioma; A, arterial anomalies; C, cardiac defects; E, eye anomalies).

Coloboma of the eyelids and lacrimal gland anomalies are associated with Tessier No. 3 clefts (oro-nasal-ocular clefts).

Macrostomia is most commonly associated with hemifacial microsomia.

2014

47
Q

A 12-year-old girl is brought to the office because of an enlarging mass of bone in the maxilla, precocious puberty, and café-au-lait spots. No aesthetic distortion or functional impact from the lesion is noted. She has intermittent bone pain in the upper extremities. Which of the following treatments is most appropriate in this patient?

A) Calcitonin
B) Doxycycline
C) Pamidronate
D) Prednisone
E) Radiation therapy

A

The correct response is Option C.

The patient described has McCune-Albright syndrome. It is a triad of polyostotic fibrous dysplasia, precocious puberty, and café-au-lait spots. Surgical intervention is not indicated in asymptomatic lesions. The general indications for surgery are aesthetic imbalance, facial disfigurement, distortion of functional occlusion, orbital dystopia, ocular proptosis, impingement on neural foramina, impingement on the optic nerve, and intractable pain. It is debatable whether contour reduction or resection and replacement of the afflicted bone is preferable. Recent literature seems to favor the latter. Biopsy is generally not needed for diagnosis, as imaging studies are generally specific for fibrous dysplasia. Equivocal imaging may warrant biopsy to confirm the diagnosis.

In general, medical treatment has had little impact on fibrous dysplasia. Early attempts at treatment included chemotherapy, glucocorticoids, calcitonin, and radiation therapy, which were all unsuccessful.

Radiation therapy should never be used, as it clearly promotes sarcomatous degeneration of fibrous dysplasia.

Pamidronate, a bone resorption-inhibiting bisphosphonate, has been shown in multiple small studies to decrease pain associated with fibrous dysplasia and decrease bone turnover. Nonsteroidal anti-inflammatory drugs can be effective, as can narcotic analgesics in pain control. Referral to a pain specialist may be necessary.

2014

48
Q

A 10-month-old male infant with 22q11.2 deletion and preoperative basic metabolic profile within the reference range is scheduled for palatoplasty to treat a soft palate cleft. After surgery, which of the following laboratory studies is most appropriate within the first 6 hours?

A) Fluorescent in situ hybridization
B) Measurement of serum phosphate concentration
C) Measurement of serum calcium concentration
D) Measurement of serum potassium concentration
E) Measurement of serum sodium concentration

A

The correct response is Option C.

Patients with 22q11.2 deletion may have cardiac abnormalities, renal issues, immune deficiencies, speech and feeding delays, mental health issues, developmental delay, cleft palate, and calcium regulation disturbances. Postoperatively, patients with 22q11.2 deletion may be at higher risk than non-deletion patients for hypocalcemia and should have postoperative calcium concentration checked in the first 6 hours postoperatively to identify and correct any abnormalities. Failure to identify postoperative hypocalcemia may lead to increased morbidity and mortality. Fluorescent in situ hybridization is a genetic test that is unnecessary in this patient who already has the diagnosis of 22q11.2 deletion. The sodium, potassium, and phosphate values are not routinely obtained early postoperatively.

2014

49
Q

A newborn in the neonatal intensive care unit is small for gestational age and has choanal atresia, cryptorchidism, a ventricular septal defect, and abnormal external ear framework. Prenatal findings included growth retardation with poor fetal movement. Which of the following additional characteristics is most likely in this newborn?

A) Bicoronal craniosynostosis
B) Coloboma
C) Radial hypoplasia
D) Syndactyly
E) Webbing of the neck

A

The correct response is Option B.

The newborn described has CHARGE (coloboma of the eye, heart defects, atresia of the nasal choanae, retardation of growth and/or development, genital and/or urinary abnormalities, and ear abnormalities and deafness) syndrome. Bicoronal craniosynostosis is associated with Crouzon, Apert, and Pfeiffer syndromes, which do not include all of the other defects. Radial hypoplasia is associated with VACTERL (vertebral defects, anal atresia, cardiac defects, tracheo-esophageal fistula, renal anomalies, and limb anomalies) syndrome. Syndactyly can be seen in many conditions including Down syndrome, Apert syndrome, and Carpenter syndrome. Webbing of the neck can be commonly seen in Noonan syndrome, Klippel-Feil syndrome, and Turner syndrome.

2014

50
Q

A 3-year-old boy is brought to the office because of abnormal head shape since birth. Photographs are shown. The patient is developmentally appropriate and has no other medical problems. Which of the following is the most appropriate surgical procedure for correction of this deformity?

A) Bilateral fronto-orbital advancement
B) Endoscopic craniotomy and helmet therapy
C) Monobloc distraction
D) Spring-mediated cranioplasty
E) Total vault reconstruction

A

The correct response is Option E.

The patient in the photograph has the classic features of sagittal suture synostosis. The primary clinical features of scaphocephaly are lengthening of the cranial vault in the anterior-posterior dimension, an anteriorly displaced cranial vertex, bullet-shaped occiput, biparietal and/or temporal narrowing and frontal bossing. All of these features are present in this patient. Because of the patient’s late clinical presentation for treatment, the most appropriate surgical procedure would be a total cranial vault reconstruction of which there are many variations and techniques.

Because of the patient’s age, he is not a candidate for either endoscopic-assisted wide strip craniotomy or spring-mediated cranioplasty. Ideal candidates for either procedure are ideally under age 6 months and 9 months, respectively, for an adequate clinical result. The results of both techniques have been shown to be comparable to traditional open remodeling procedures and are generally less invasive surgical procedures.

Although the monobloc distraction may allow the opportunity to remodel the forehead, there is no clinical indication for midface distraction in the patient described. A monobloc distraction alone will not correct the other abnormal features of scaphocephaly. It is not an indicated procedure for the correction of isolated sagittal suture synostosis.

Bilateral fronto-orbital advancement may allow the opportunity to remodel the forehead, but it will not address the other cranial vault abnormalities. There is no indication for remodeling the supraorbital bar in this case.

2014

51
Q

A 6-month-old female infant presents with a wide, tall forehead, low-set ears, and supraorbital rim retrusion. CT scan demonstrates bicoronal synostosis. Genetic testing is positive for TWIST mutation. Which of the following additional findings is/are characteristic of this patient’s disorder?

A) Cervical spine anomalies
B) Complete cartilaginous tracheal rings
C) Eyelid ptosis
D) Gastroschisis
E) Thumb duplication

A

The correct response is Option C.

The patient described has Saethre-Chotzen syndrome as confirmed by bilateral coronal synostosis, low-set ears, and mutations of the TWIST gene. In addition to these findings, patients with Saethre-Chotzen syndrome often have eyelid ptosis, which is a distinguishing feature from other forms of syndromic craniosynostosis. It is important to recognize ptosis in infants and young children in order to maintain adequate visual pathway development. Thumb duplication is not found in patients with Saethre-Chotzen syndrome. Tracheal anomalies are associated with Pfeiffer syndrome. Cervical spine anomalies can be found in both Apert and Pfeiffer syndromes. Gastroschisis is not associated with syndromic craniosynostosis.

2019

52
Q

A 4-year-old boy is referred to the clinic for evaluation. Medical history includes a cardiac defect requiring surgery, submucous cleft palate, hypernasality, and developmental delay. His parents report that he undergoes cardiology follow-up annually. Further testing is most likely to detect an abnormality that will require consultation with which of the following specialties?

A) Endocrinology
B) Gastroenterology
C) Immunology
D) Nephrology
E) Psychiatry

A

The correct response is Option C.

The patient described has 22q11.2 deletion syndrome (formerly known as velocardiofacial syndrome or DiGeorge syndrome). This syndrome is the most common chromosomal deletion error in fetuses, with a prevalence of 1 in 3000 to 6000 live births. 22q11.2 Deletion syndrome is a common cause of hypernasality. Children with congenital heart defects and hypernasality should be worked up for 22q11.2 deletion syndrome. Either a FISH probe or microarray can detect the chromosomal deletion that occurs in the LCR22A–LCR22D region of the chromosome.

Children with 22q11.2 deletion syndrome can present with a myriad of clinical manifestations. The most common clinical manifestation is a congenital cardiac defect, particularly of the outflow tracts (e.g., tetralogy of Fallot). Congenital cardiac disease remains the primary cause of mortality in this patient population.

Hypernasality is another common finding within this patient population, occurring in approximately 65% of patients with 22q11.2 deletion syndrome. Classic workup for this involves imaging of the velopharyngeal mechanism (either nasopharyngoscopy or video fluoroscopy) and imaging of the posterior pharyngeal pharynx with MRI and evaluation of aberrant/medialization of the internal carotid arteries.

Immunologic abnormalities are the most common of the group. A referral to immunology should be initiated in all patients with 22q11.2 deletion syndrome since up to 75% of this patient population can have thymic hypoplasia and diminished T cell production. Children with 22q11.2 deletion syndrome can have chronic infections and poor responses to vaccinations.

Gastrointestinal conditions such as poor feeding, gastroesophageal reflux, and vomiting or constipation occur in approximately 30% of patients with 22q11.2 deletion syndrome. More concerning GI conditions such as malrotation or tracheoesophageal fistula have been found in patients with this syndrome.

Hypocalcemia secondary to hypoparathyroidism can present as tetany, seizures, or feeding issues. Hypocalcemia presents in approximately 50 to 65% of patients with 22q11.2 deletion syndrome. Thyroid function can also be abnormal with hypothyroidism possible.

Nephrology consultation should be considered if abdominal ultrasonography shows renal agenesis, duplication of the collecting system, or cystic kidney disease. About 33% of patients with 22q11.2 deletion syndrome have some abnormality related to the genitourinary system.

Patients with 22q11.2 deletion syndrome are at increased risk for psychiatric disorders such as attention-deficit/hyperactivity disorder (ADHD), autism spectrum disorder, and anxiety (particularly when they progress into adulthood). The rate of schizophrenia is increased in this patient population compared with unaffected individuals.

2019

53
Q

Patients with which of the following conditions have the highest incidence of accessory auricle?

A) Apert syndrome
B) Goldenhar syndrome
C) Treacher Collins syndrome
D) VACTERL association
E) Van der Woude syndrome

A

The correct response is Option B.

In a recent review of the literature about congenital accessory auricle, patients with Goldenhar syndrome were found to have an accessory auricle reported 100% of the time.

Of the options listed, VACTERL (Vertebral, Anal, Cardiac, Tracheal, Esophageal, Renal, and Limb association) has the lowest association with an accessory auricle at 16%. Patients with VACTERL typically are found to have vertebral defects, anal atresia, cardiac anomalies, tracheoesophageal fistula, renal anomalies, and limb differences.

Patients with Treacher Collins syndrome are found to have an accessory auricle present 30 to 40% of the time. These patients often have mandibular hypoplasia, zygomatic hypoplasia, external ear anomalies, conductive hearing loss, eyelid colobomas, cleft palate, and dental anomalies. It is associated with several different genes, including TCOF1, POLR1C, and POLR1D.

Patients with Apert syndrome and Van Der Woude syndromes do not typically have accessory auricles.

2019

54
Q

A 6-year-old girl is referred for velopharyngeal dysfunction and on physical examination is found to have a submucous cleft and bifid uvula. Which of the following additional clinical findings is most likely to support the diagnosis of 22q11.2 deletion syndrome?

A) Hypocalcemia
B) Hypothyroidism
C) Microtia
D) Overjet
E) Retinal detachment

A

The correct response is Option A.

22q11.2 Deletion syndrome (also referred to as DiGeorge syndrome, velocardiofacial syndrome, and CATCH-22) is a congenital disorder caused by the deletion of a segment of chromosome 22. Symptoms of this syndrome often include anomalous carotid arteries and conotruncal cardiac abnormalities such as truncus arteriosus and tetralogy of Fallot. The prevalence of atrial septal defect (ASD) has been reported to be 12% in patients with velocardiofacial syndrome. Other symptoms and findings include absent or hypoplastic thymus, developmental delay, cleft palate, and hypocalcemia related to hypoparathyroidism. Patients with velocardiofacial syndrome also have abnormal facial features including a broad nasal root, low-set ears, retrognathia, elongated face or maxillary excess, and epicanthal folds. Overjet and retinal detachment are not associated with velocardiofacial syndrome. Hypothyroidism is not as common as hypocalcemia in this patient population.

2019

55
Q

An 18-month-old child is brought to the office after undergoing fronto-orbital advancement for metopic craniosynostosis. Which of the following is the earliest age the surgeon can order the x-ray studies and expect to be able to see frontal sinus development?

A) 1 year
B) 2 years
C) 4 years
D) 6 years
E) 10 years

A

The correct response is Option D.

Frontal sinus development is associated with specific age-related periods of growth of the skull. The frontal sinus is absent at birth and during the initial phase of growth of the skull. The sinus is visible only in x-ray studies at the end of the first period of skull growth. This is the time when the endocranial table of the skull ceases to grow and conforms to the general shape of the brain. This is not seen on x-ray studies until 6 years of age or 72 months.

2019

56
Q

A female infant is born with severe Treacher Collins syndrome and bilateral Pruzansky III mandible (absence of condyle). Tracheostomy is performed for respiratory distress. Which of the following surgeries is most likely to allow decannulation?

A) Alloplastic condylar reconstruction
B) Bilateral sagittal split osteotomy
C) Costochondral rib grafts
D) Mandibular distraction
E) Tongue-lip adhesion

A

The correct response is Option C.

Patients with Treacher Collins syndrome may have a varied presentation. The mandible hypoplasia may be mild or severe. In this case the patient has no temporomandibular fossa or condyles. Tongue-lip adhesion and mandibular distraction are used in severe Pierre Robin sequence. Because of the absence of condyles and temporal mandibular joints, distraction, and sagittal split osteotomy are not the best options. A costochondral graft will provide better airway support and can also be distracted in the future.

2019

57
Q

Which of the following statements is correct about Tessier clefts No. 3, No. 4, and No. 5?

A) Tessier No. 3 involves the alveolar ridge, while Tessier No. 5 does not
B) Tessier No. 3 is medial to the infraorbital nerve, while Tessier No. 4 is lateral
C) Tessier No. 3 only affects the oral region, while Tessier No. 4 only affects the orbital region
D) Tessier No. 4 involves the piriform aperture, while Tessier No. 5 does not
E) Tessier No. 4 is medial to the infraorbital nerve, while Tessier No. 5 is lateral

A

The correct response is Option E.

Tessier No. 3 and No. 4 are medial to the infraorbital nerve, but Tessier No. 5 is lateral.

Tessier No. 3 involves clefts of the nose, orbit, and lip (naso-oral-ocular cleft), whereas Tessier No. 4 involves the lip and orbit (oral-ocular cleft), and the nose is uninvolved.

Tessier No. 5 involves oral, cheek (maxillary sinus), and orbital cleft and is the rarest.

2019

58
Q

Which of the following is the most common cause of scaphocephaly without craniosynostosis?

A) Fibromatosis colli
B) Hereditary dolichocephaly
C) Ocular torticollis
D) Prematurity
E) Ventriculoperitoneal shunt

A

The correct response is Option D.

Positional plagiocephaly is very common since the American Academy of Pediatrics began the ‘back to sleep’ campaign to decrease sudden infant death syndrome in 1992. Occipital flattening is the most common type seen. Positional plagiocephaly risk factors include prematurity, intrauterine position, congenital muscular torticollis, plural birth, motor delays, and hypotonia. Ventriculoperitoneal (VP) shunts may induce a secondary craniosynostosis if over-shunting occurs. Scaphocephaly head shape without craniosynostosis is noted in the premature babies in the NICU due to positioning and tone. This corrects with growth, increased tone, and repositioning. Presence of a VP shunt does not lead to the head shape of scaphocephaly without craniosynostosis but may be seen in children after intraventricular hemorrhage in the NICU. Scaphocephaly without craniosynostosis does occur in older infants and children but is not known to be a hereditary condition. Both ocular torticollis and congenital muscular torticollis can cause a head tilt and are associated with deformational plagiocephaly. Fibromatosis colli is seen with severe muscular torticollis and is associated with plagiocephaly.

2019

59
Q

A 2-day-old female newborn is evaluated in the neonatal intensive care unit. Physical examination shows epicanthal folds, a flat nasal bridge, and a wide U-shaped cleft palate. She has micrognathia with 12 mm of overjet, substernal and costal retractions and desaturations while supine, and is not able to feed orally. Ophthalmologic evaluation shows bilateral cataracts. Which of the following metabolic abnormalities is most likely responsible for this patient’s condition?

A) Abnormal cellular response to fibroblast growth factor
B) Abnormal formation of collagen II
C) Abnormal regulation of craniofacial bone and suture embryogenesis
D) Dysregulation of embryogenesis caused by a multi-gene deletion on chromosome 22
E) Inadequate production of surfactant by the lungs

A

The correct response is Option B.

The neonatal patient described in the vignette has clear signs of respiratory distress in the setting of micrognathia and glossoptosis, also known as Pierre Robin sequence (PRS). PRS can be isolated or associated with a broad metabolic disturbance, the most common of which is Stickler syndrome. Stickler syndrome is a disruption of the formation of collagen, which can lead to multiple congenital anomalies including a flat nasal bridge, hearing loss, cleft palate, limb anomalies, micrognathia, and ophthalmologic issues including cataracts.

Mutations in the fibroblast growth factor receptor can lead to syndromic forms of craniosynostosis such as Apert, Crouzon, or Pfeiffer syndromes. Disruption of suture embryogenesis is caused by a mutation in the TWIST1 gene and gives rise to Saethre-Chotzen syndrome. Deletion of the small arm of chromosome 22 leads to 22q11.2 deletion syndrome, previously known as DiGeorge syndrome or Velocardiofacial syndrome. Lack of lung surfactant is most often caused by prematurity and is not associated with PRS.

2020

60
Q

A 5-year-old girl has craniosynostosis, a low-set hairline, ptosis, and 4th/5th-digit syndactyly of both hands. Genetic testing is most likely to show a mutation in which of the following genes?

A) EFNB1
B) FGFR2
C) FGFR3
D) MPDZ1
E) TWIST1

A

The correct response is Option E.

The clinical picture is consistent with Saethre-Chotzen syndrome. It is an autosomal dominant condition defined by a genetic mutation or deletion affecting the TWIST1 gene or chromosome 7p21. FGFR2 mutations are predominantly associated with Apert, Crouzon, and Pfeiffer syndromes. FGFR3 mutations are associated with Muenke syndrome and Crouzon syndrome with acanthosis nigricans. EFNB1 is associated with craniofrontonasal syndrome. MPDZ1 is associated with hydrocephalus.

2020

61
Q

A 2-year-old boy presents with swelling over the bridge of the nose that has been present since birth. The swelling has been slowly increasing in size and he has hypertelorism. The swelling is soft, compressible, and it transilluminates. There are visible and palpable pulsations, and the mass enlarges when the patient cries. Which of the following is the most likely diagnosis?

A) Encephalocele
B) Glioma
C) Hemangioma
D) Nasal dermoid cyst
E) Nasopharyngeal angiofibroma

A

The correct response is Option A.

Encephaloceles are neural tube defects that result in sac-like protrusions of the meninges (meningocele) or brain and meninges (meningoencephalocele) in various locations along the cranium, such as between the forehead and nose (including naso-orbital, frontonasal, and nasoethmoidal locations) or on the back of the skull. They tend to be soft, compressible masses that transilluminate that may be sessile or pedunculated. Biopsy may result in a cerebrospinal fluid leak.

Glioma is a mass of ectopic neural tissue that does not transilluminate.

Hemangiomas are benign vascular lesions that are present at birth and characterized by a rapid growth phase around the age of 1 to 6 months, followed by gradual involution over 1 to 12 years. They have no intracranial connection and no cerebral pulsations. Nasopharyngeal angiofibromas, also known as juvenile nasopharyngeal angiofibromas, are benign but locally invasive vascular tumors that occur almost exclusively in adolescent males. They present with unilateral or bilateral nasal obstruction, frequent epistaxis or blood-tinged nasal discharge. Nasal dermoid cyst is a benign cystic lesion that does not pulsate and does not transilluminate.

2020

62
Q

A 23-year-old woman with severe progressive hemifacial atrophy that has been stable for 3 years now desires a long-term stable reconstruction. Which of the following is the most appropriate recommendation for reconstruction of this patient’s facial asymmetry?

A) Alloplastic bony augmentation
B) Contralateral suction lipectomy
C) Delay reconstruction until it has been stable for 10 years
D) Free tissue transfer
E) Hyaluronic acid injections

A

The correct response is Option D.

Progressive hemifacial atrophy is also known as Parry-Romberg syndrome. The progression is ultimately self-limiting. Reconstruction 2 years or more after burn out is commonly accepted. For very mild asymmetry, hyaluronic acid fillers can improve symmetry, but require recurrent treatments. For mild to moderate asymmetry, microfat grafting can restore symmetry. Multiple sessions may be required to achieve long-term correction. For severe asymmetry, free muscle flap with parascapular flap or anterolateral thigh flap can provide enough soft tissue bulk for long-term correction.

Alloplastic bony augmentation would correct any potential bony deficiencies, but would not address any soft tissue deficiencies.

2020

63
Q

A 15-year-old girl has a 12-month history of pain and fullness in the right supraorbital rim. She has café-au-lait spots, a history of precocious puberty, and a recent pathologic rib fracture. Which of the following is the most likely pathology of the lesion?

A) Dermoid cyst
B) Fibrous dysplasia
C) Neurofibroma
D) Osteoblastoma
E) Rhabdomyosarcoma

A

The correct response is Option B.

This patient has a classic presentation of McCune-Albright syndrome. Patients with McCune-Albright present with a triad of polyostotic fibrous dysplasia, precocious puberty, and skin pigmentation (eg, café au lait spots). Additionally, they may have hyperfunctioning endocrinopathies such as growth hormone excess. If these patients present with intramuscular myxomas, it is known as Mazabraud syndrome. Malignant degeneration of fibrous dysplasia has been reported in up to 4% of patients with McCune-Albright syndrome. Management depends on the clinical presentation and functional impact of the lesions, and is primarily surgical.

Patients with neurofibroma may have café au lait spots, but not precocious puberty or pathologic fractures. Dermoid cysts generally do not present with pain and are often noted at a much younger age than the patient described.

2020

64
Q

A 14-year-old girl with Crouzon syndrome presents with a severe Angle Class III malocclusion, mid face retrusion, and severe sleep apnea. She is scheduled to undergo Le Fort III advancement using distraction osteogenesis. The risk for complications with this procedure is closest to which of the following?

A) 5%
B) 20%
C) 40%
D) 60%
E) 80%

A

The correct response is Option B.

There are several important advantages of distraction osteogenesis for Le Fort III advancement versus conventional single-stage advancement with bone grafting and these include: less regression, greater advancement distance, and no need for bone grafting. Le Fort III distraction is not without its issues. Major and minor complications have been reported in nearly 20% of patients undergoing this procedure; these complications include bone loss, pin migration, loss of fixation, meningitis, seizures, and cerebrospinal fluid leaks. Several recent reports show that these complications occur in approximately 20% of cases.

2020

65
Q

Which of the following genetic mutations is most likely to be found in a patient with orofacial clefting and popliteal pterygium?

A) Gain-of-function mutation in FGFR2
B) Gain-of-function mutation in NF1
C) Gain-of-function mutation in PIK3CA
D) Loss-of-function mutation in IRF6
E) Loss-of-function mutation in TCOF1

A

The correct response is Option D.

IRF6 mutations that result in loss-of-function have been reported in both syndromic Van der Woude syndrome as well as nonsyndromic orofacial clefting; Van der Woude syndrome can include popliteal pterygium.

PIK3CA gain-of-function mutations have been reported with venous malformations and lymphatic malformations. Mutations in TCOF1 are associated with Treacher Collins syndrome; these patients may have cleft palate, but they do not have lip pits. Gain-of-function mutations in FGFR2 have been implicated in syndromic craniosynostosis (Apert syndrome, Crouzon syndrome, and others). Mutations in NF1 result in increased RAS/MAPK signaling and neurofibromatosis type 1.

2020

66
Q

A 2-month-old infant is referred for evaluation because he has an abnormal head shape. Physical examination shows low-set ears; short, webbed fingers; and duplicate great toes. A CT scan shows sagittal and lambdoid synostosis. A mutation in which of the following genes is most likely responsible for these findings?

A) FGFR1
B) FGFR2
C) FGFR3
D) RAB23
E) TWIST1

A

The correct response is Option D.

This child has Carpenter syndrome. This syndrome is caused by a mutation in the RAB23 gene, which is located on chromosome 6. Carpenter syndrome is inherited in an autosomal recessive manner, but it can also be caused by de novo mutation in RAB23. In addition to synostosis, symbrachydactyly and preaxial polydactyly are found in patients with Carpenter syndrome.

Mutations in the other genes listed are all associated with syndromic craniosynostoses. Fibroblast growth factor receptor (FGFR) mutations have been associated with several differing syndromes: FGFR1 mutations cause Pfeiffer syndrome, FGFR2 mutations cause Apert and Crouzon syndromes, and FGFR3 mutations cause Muenke syndrome. A mutation of the TWIST1 gene causes Saethre-Chotzen syndrome.

2020

67
Q

A 15-year-old girl with a history of facial asymmetry reports worsening of her deformity in the past 3 years. It has slowly progressed with upper lip and cheek atrophy, mid-face hyperpigmentation, nasal deviation, and worsening occlusal cant. Photographs are shown. The patient reports migraine but denies any other medical problems. Which of the following is the most likely diagnosis?

A) Fibrous dysplasia
B) Goldenhar syndrome
C) Neurofibromatosis
D) Parry-Romberg syndrome
E) Systemic scleroderma

A

The correct response is Option D.

Parry-Romberg syndrome is a rare craniofacial disease characterized by slow progressive hemifacial atrophy of the skin, subcutaneous tissue, and, in severe cases, bony structures, following ipsilateral branches of the trigeminal (V) nerve. It usually occurs in the first two decades of life, and it is more prevalent in females. It is frequently associated with neurologic and ophthalmologic involvement. The severity of deformity varies depending on the age of onset of disease, being more severe the earlier it starts.

2022

68
Q

A 3-month-old infant presents with scaphocephaly and sagittal suture ridging. Spring-assisted cranioplasty with an endoscopic approach is planned. Which of the following is an advantage of this procedure over open cranial vault remodeling procedures?

A) Decreased need for additional surgeries
B) Greater control of bony movements
C) Improved aesthetic outcomes
D) Less blood loss
E) Lower risk for developing neurocognitive impairment

A

The correct response is Option D.

Spring-assisted cranioplasty or spring-assisted synostosis surgery is a technique to address sagittal craniosynostosis. This procedure is similar in approach to suturectomy/helmet procedures, which involve removal of the fused suture via small incisions, often with the assistance of an endoscope. In spring-assisted cranioplasty, however, no helmet is applied to the skull to reshape it. Rather, two to three stainless steel springs are placed between the now separated parietal bones that are widened over time as the springs expand. These springs need to be removed around 4 months later in a short second operation. Compared with more traditional open cranial vault reconstruction procedures, spring-assisted cranioplasty is associated with decreased blood loss, shorter operative time (even including the second surgery) and hospital stay, and comparable aesthetic and neurocognitive outcomes. The surgeon is, however, compromising control of the postoperative outcome in favor of the smaller surgical footprint and the guaranteed second surgery required to remove the springs.

2022

69
Q

Which of the following best describes the growth pattern of the calvarium relative to a fused suture?

A) Oblique
B) Parallel
C) Perpendicular
D) No relation

A

The correct response is Option B.

Virchow’s law states that bone surrounding a prematurely fused suture only grows parallel to the suture, while growth perpendicular to the suture is restricted. This contributes to the characteristic patterns of cranial shape in the various types of synostoses. Sagittal fusion yields a skull that is long in the anteroposterior direction and narrow bitemporally. Metopic fusion yields a triangular-shaped skull due to constriction caused by the anterior, medial metopic suture. Unilateral coronal synostosis yields an asymmetric skull, notably for a trapezoidal appearance from the overhead inspection. While there may be some ridging above and below the affected suture, this does not represent the growth of normal calvarial bone.

2022

70
Q

A 3-month-old infant presents with flattening of the left forehead, and asymmetry of the orbits and face. He has early evidence of right-sided astigmatism and ocular torticollis. A CT scan is shown. The parents are considering two options: endoscopic suturectomy and helmet therapy at age 3 months or fronto-orbital advancement (FOA) at age 9 months. Compared with endoscopic suturectomy and helmet therapy, FOA is more likely to result in which of the following surgical outcomes?

A) Greater need for revision
B) Greater postoperative facial symmetry
C) Less operative morbidity
D) Lower overall cost
E) More severe strabismus

A

The correct response is Option E.

Surgical intervention is recommended for most patients with craniosynostosis due to the variable risk for increased intracranial pressure (ICP), localized cerebral compression, and developmental delay. Traditional treatment techniques such as fronto-orbital advancement (FOA) and total cranial vault remodeling are still widely used, but minimally invasive options such as endoscopic suturectomy and postoperative helmet therapy (ES + HT), spring-mediated distraction, and conventional distraction have become more commonplace. While each technique has its ardent supporters, there is a growing number of studies that compare the clinical outcomes of each technique. ES + HT is most effective when performed under 4 months of age because the correction depends solely on cranial expansion, while spring-mediated distraction is typically done in older infants (4 to 8 months of age). Cranial remodeling has no strict age limit, but many surgeons defer treatment until 7 to 10 months to decrease the risk associated with anesthesia and recurrence of the malformation. Despite these concerns, there are some data that support early cranial expansion (<6 months of age) to optimize neurocognitive outcomes. Compared with FOA, endoscopic and spring-mediated distraction techniques are less costly, less morbid, and have similar revision.

The patient has left unilateral coronal craniosynostosis (UCS), which manifests as ipsilateral forehead and brow retrusion and shortening of the ipsilateral hemi-cranium, with orbital and facial asymmetry. Open cranial vault procedures do not directly correct the facial symmetry and at least two investigations show better facial asymmetry is achieved with early, minimally invasive approaches. The orbital asymmetry results in strabismus (contralateral head tilting termed ocular torticollis) and contralateral astigmatism in the majority of patients. Several studies confirm significantly better ophthalmologic outcomes following early endoscopic suturectomy for UCS versus FOA performed at an older age.

2022

71
Q

A 7-year-old girl presents to the clinic with poor school performance, speech difficulties, and hypernasality. The patient also has a history of immunodeficiency and hypocalcemia. Clinically, there is no evidence of overt or submucosal cleft palate. Which of the following is the most appropriate next step for her care?

A) Formal speech evaluation
B) MRI
C) Palatoplasty
D) Observation

A

The correct response is Option A.

Velopharyngeal dysfunction can be divided into insufficiency, incompetence, and mislearning. The term insufficiency is related to a structural defect that results in poor closure of the velopharyngeal port. Incompetence is usually associated with neurological or muscular causes to inefficient port closure. Mislearning is less frequent and related to normal anatomy and muscle function, but inadequate production of sounds.

The patient described has many of the medical abnormalities observed in 22q11.2 deletion syndrome. These include, but are not limited to, congenital heart defects, hypocalcemia, immune deficiencies, speech delay/disorders, palatal abnormalities, intellectual disabilities, and psychiatric disorders. Late speech disorder diagnosis is not uncommon and warrants a thorough evaluation in a cleft center; this patient would also benefit from a genetics consultation. Depending on the formal speech evaluation results, further measures of velopharyngeal closure, such as nasometry and pressure flow, as well as imaging with videofluoroscopy and nasopharyngoscopy, might be recommended.

2022

72
Q

A 4-month-old infant is evaluated because of a lesion near the lateral aspect of the left eyebrow. A photograph is shown. The mass is firm, well-circumscribed, and does not move with overlying skin movement. Which of the following preoperative imaging studies is most appropriate?

A) CT scan
B) MRI
C) Ultrasonography
D) X-ray studies
E) Imaging studies are not necessary

A

The correct response is Option E.

Dermoid cysts of the craniofacial region represent about 7% of all dermoid cysts and have an incidence of around 1 in 1000 live births. These masses can arise in multiple anatomic locations on the head, including the lateral eyebrow (sometimes referred to as an angular dermoid), midline nose, glabella, medial canthus, temporal or occipital scalp, frontal/anterior fontanel, and postauricular sulcus. Intracranial extension occurs in midline nasal dermoid cysts (13% below and 0% above the nasofrontal suture), frontal/fontanel (36.8%), temporal (16.2%), and occiput (16.3%), while this almost never occurs in the remaining locations (lateral brow, parietal, medial canthus, auricular). Cysts of the lateral brow represent almost two-thirds of all craniofacial dermoid cysts, and intracranially, extension or contact with the dura does not occur. Thus, the diagnosis is clinical, and further imaging in a child this young is not warranted. Overland et al. advocate imaging (CT or MRI) in older patients (>2 years of age) due to an increased risk for orbital extension with growth.

2022

73
Q

A 3-month-old male infant presents with left occipital flattening and associated external ear and facial asymmetry. Significant head tilt and limited range of motion with neck rotation are noted on physical examination. A photograph is shown. Which of the following muscles is most likely contributing to the patient’s plagiocephaly?

A) Digastric
B) Platysma
C) Splenius cervicis
D) Sternocleidomastoid
E) Trapezius

A

The correct response is Option D.

Congenital muscular torticollis occurs as a result of sternocleidomastoid shortening, presenting as head tilt and limited range of motion with neck rotation. The inability to rotate the neck increases the time spent in the preferred position and contributes to deformational plagiocephaly. The digastric muscle acts to elevate the hyoid, while the platysma is involved in facial expression. The splenius cervicis extends the spinal column, and the trapezius functions to maintain posture and stabilize the scapula.

2022

74
Q

A 5-year-old boy has an interrupted aortic arch, low-set ears, broad nasal bridge, velopharyngeal insufficiency, and hypoparathyroidism. A photograph is shown. Which of the following additional findings are most likely in this patient?

A) Facial asymmetry, microtia
B) High myopia, retinal detachment
C) Lower eyelid coloboma, downward-slanting palpebral fissures
D) Lower lip pits, salivary mucous drainage
E) T-cell immunodeficiency, chronic otitis media

A

The correct response is Option E.

22q11.2 Deletion syndrome (also known as DiGeorge, velocardiofacial, and conotruncal anomaly face syndromes) is often identified with help of the “CATCH 22” mnemonic, which refers to its characteristics of cleft palate, abnormal facies, thymic hypoplasia, cardiac malformations, and hypoparathyroidism. T-cell immunodeficiency and recurrent otitis media infections occur secondary to thymic hypoplasia and cleft-related ear anomalies.

Van der Woude syndrome is characterized by cleft palate and/or cleft lip, and lower lip pits/fistulae. Abnormal salivary gland morphology and hypodontia may also occur.

Treacher Collins syndrome (mandibulofacial dysostosis) is characterized by malar and mandibular hypoplasia, down-slanting palpebral fissures, lower eyelid coloboma, and ear anomalies.

Stickler syndrome type 1 is characterized by cleft palate, facies such as mid-face hypoplasia/retrusion, myopia with increased risk for retinal detachment, sensorineural hearing loss, and joint abnormalities including hypermobility and early-onset osteoarthritis.

Hemifacial microsomia is characterized by small and/or flattened maxillary, temporal, and zygomatic bones. Facial asymmetry is further apparent upon animation when soft tissue and nerve hypoplasia occur. Oral clefts and ear anomalies are commonly involved, as well as underdevelopment of the orbits.

Van der Woude syndrome, Treacher Collins syndrome, Stickler syndrome, and hemifacial microsomia are all unrelated to hypoparathyroidism.

2022

75
Q

The ear anomaly depicted in the photograph shown is most commonly found in patients with which of the following syndromes?

A) Apert
B) Klippel-Trenaunay
C) Stickler
D) Treacher Collins
E) Van der Woude

A

The correct response is Option D.

The photograph depicts a patient with lobular type microtia and represents failure of formation of the pinna of the ear. Patients commonly have stenosis or absence of the external auditory canal, and in cases of lobular type microtia, exhibit a microtic remnant composed of malformed cartilage and fibrofatty tissue that is reminiscent of the ear lobule. This condition may be seen in isolation but is commonly seen in cases of hemifacial microsomia and Treacher Collins syndrome. Treacher Collins syndrome, or mandibulofacial dysostosis, exhibits a broad phenotypic spectrum and is commonly seen with hypoplasia or absence of the zygoma, coloboma of the lower eyelid, lateral canthal dystopia, absence of the medial lower eyelid lashes, mandibular hypoplasia with a steep occlusal plane, and abnormalities of the external ear, including microtia.

Apert syndrome results from FGFR2 mutations Ser252Trp or Pro253Arg and results in multisuture craniosynostosis, mid-face hypoplasia, complex syndactyly, cystic acne, and other less common findings. It is transmitted as an autosomal dominant condition.

Stickler syndrome is inherited in autosomal dominant and recessive patterns. It results from mutations in collagen genes COL2A1, COL11A1, and COL11A2. This may manifest with ocular abnormalities including early-onset cataracts, vitreous anomalies, retinal detachments, and severe myopia. Patients may have sensorineural hearing loss. Mandibular hypoplasia and Pierre Robin sequence is common in these patients.

Van der Woude syndrome results from a mutation of the IRF6 gene and is a common autosomal dominant condition associated with cleft lip and palate. Patients characteristically exhibit lower lip pits.

Finally, Klippel-Trenaunay syndrome is a congenital capillary-lymphatic-venous malformation of the lower extremity with associated soft tissue and bony overgrowth. This condition is caused by a somatic mutation in the PIK3CA gene and is therefore on the phenotypic spectrum with CLOVES syndrome and other PIK3CA-related overgrowth syndromes (PROS).

2022

76
Q

A 9-year-old boy is brought by his parents to the clinic for evaluation of a lesion on the right cheek. The lesion has steadily grown since it first appeared 6 months ago. On examination, the lesion has overlying bluish skin discoloration with an associated mobile, firm, and well-defined subcutaneous mass. Excisional biopsy is performed, and pathology results show a calcified benign mass. Which of the following is the most likely diagnosis?

A) Lipoma
B) Nevus sebaceous
C) Pilar cyst
D) Pilomatrixoma
E) Sebaceous cyst

A

The correct response is Option D.

Pilomatrixomas, also known as calcifying epitheliomas, are benign adnexal tumors arising from the hair follicle matrix cells. They are common in the pediatric population and often arise in the mid face region of the head and neck. They are typically nonresolving lesions that slowly grow and are firmer in comparison with inclusion or sebaceous-type cysts. Excisional biopsy is the preferred treatment with low recurrence rates (1.5%). Spontaneous regression has not been reported.

A nevus sebaceous is a skin lesion without a mass component that is classically salmon in color. These lesions are epidermal in origin and have the potential to become basal cell carcinomas. Sebaceous cysts are common cystic lesions associated with sebum collecting within a cystic mass. They slowly grow and often do not resolve, but they lack the calcified, firm component seen in pilomatrixoma.

Lipomas are benign hypertrophy of adipose cells. They are typically well-defined subcutaneous masses without overlying skin changes. They are spongy, soft tumors that are typically present for 1 or more years before presentation. Pilar cysts are firm, mobile masses that typically present in the hair-bearing scalp. They often have an inflamed overlying skin component with red discoloration. They also arise from the hair shaft and appear as firm, smooth, white nodules. Also known as trichilemmal cysts, pilar cysts are differentiated from pilomatrixoma by their homogenous firm appearance, presence in the scalp, and lack of a calcified component.

2023

77
Q

A 15-year-old girl is brought to the office for management of facial asymmetry. Over the past 2 years, the patient has progressively developed atrophy of the soft tissues of the left side of her face. She reports that the asymmetry is getting worse. Physical examination shows marked soft-tissue atrophy of the left mid face and lower face with a slight upward occlusal cant. Which of the following therapies is the most appropriate treatment for this patient’s condition?

A) Calcium carbonate
B) Clindamycin
C) Methotrexate
D) Sodium thiosulfate
E) Warfarin

A

The correct response is Option C.

This patient has progressive hemifacial atrophy (Parry-Romberg syndrome). This condition is characterized by atrophy of the hemi face typically in the mid and lower face (cranial nerve V2 and V3 distribution), although the forehead may also be involved. The skin and subcutaneous tissue are most frequently involved, although it can also affect muscle, cartilage, and bone. Onset is in the first two decades of life in the majority of cases. The disease progresses for 2 to 10 years before it stabilizes (“burns out”). Parry-Romberg syndrome is thought to be an immune-mediated disease. There is currently no cure for the condition. Methotrexate is considered one of the first-line medical treatments to halt disease progression. It is frequently combined with prednisone for the first 3 months since methotrexate has a delayed effect on inflammation and fibrosis. Current evidence suggests that methotrexate therapy be administered for 12 to 24 months to induce prolonged remission. Methotrexate is administered in oral or intravenous form, at a dosage of 0.3 to 1 mg/kg/week with a maximum dose of 25 mg/week. Immunosuppressive drugs, like cyclosporine, mycophenolate mofetil, and cyclophosphamide, have been reported as treatments in isolated case reports. Other treatments that have been described are antimalarial, retinoids, penicillamine, tetracycline, and topical steroids with variable success.

The other choices are not medications used for this condition.

2023

78
Q

A 21-month-old boy with Treacher Collins syndrome is scheduled to undergo mandibular distraction osteogenesis. Which of the following is the primary mechanism by which new bone will form for the mandibular lengthening associated with this procedure?

A) Endochondral ossification
B) Intramembranous ossification
C) Osteoconduction
D) Osteogenesis
E) Osteoinduction

A

The correct response is Option B.

Distraction osteogenesis is a technique in which an osteotomy is first performed, and then distraction of the surrounding vascularized bone segments is performed with a mechanical device. The hallmark of this process is formation of new bone within the distraction gap. This technique is commonly used for reconstruction of certain bony defects in the craniofacial skeleton.

New bone is formed during distraction osteogenesis via the process of intramembranous ossification. The key is that the new bone is spontaneously formed without a cartilaginous intermediate. In addition to distraction osteogenesis, intramembranous ossification occurs during fetal development with the formation of the flat bones. It also occurs with primary bone healing.

Osteogenesis is the primary mechanism for healing of a vascularized bone graft. Osteoblasts that survive the bone graft transfer procedure and osteoblasts at the native recipient bone both participate in the formation of new bone to heal the bony interface.

Osteoconduction is the primary process for healing of cortical bone grafts. Blood vessels and cells from the recipient wound bed grow into the cortical bone graft, which functions as a template for the deposition of new bone.

Endochondral ossification involves formation of new bone via a cartilaginous intermediate. This process is seen during fetal long bone development and during bony fracture healing via a cartilaginous callus.

Osteoinduction, which is seen in healing of cancellous bone grafts, involves bone morphogenetic protein stimulating mesenchymal cells at the recipient site to differentiate into osteoprogenitor cells.

2023

79
Q

A 5-year-old girl presents to the clinic with a right temporoparietal cranial defect. History is significant for a nonaccidental trauma requiring a decompressive craniectomy 2 years ago. The initial bone flap was replaced but has resorbed due to a prior infection. The residual defect measures 10 × 8 cm. Which of the following materials is most appropriate for reconstruction of this defect?

A) Demineralized bone
B) Hydroxyapatite cement
C) Methyl methacrylate
D) Porous polyethylene
E) Split calvarial bone

A

The correct response is Option E.

The most appropriate method of cranial reconstruction in the case of this pediatric patient is split calvarial bone graft.

Autologous split calvarial bone is the most appropriate material due to the large size of the defect, the age of the patient, and the prior history of infection. Calvarial bone can be split as early as age 3 years, when the diploic space has developed. The other materials have been used in pediatric patients but have some limitations. Demineralized bone has been shown to be osteoinductive, but the large size of the bone gap makes this method costly, and there is a higher risk for not completely ossifying the gap. Hydroxyapatite cement has a similar crystalline structure to the mineral phase of human bone and can be remodeled over time; however, this material is expensive, prone to fracture, and has a higher complication rate with regards to osseointegration, infection, and extrusion. Methyl methacrylate is an acrylic that is often used in nonautologous adult cranioplasties since it is biocompatible and has similar hardness to natural bone; however, the risk for infection is much higher and it causes a highly exothermic reaction during the setting process that can lead to complications. Porous polyethylene is a ceramic that can also be used for large defects, but it is more expensive, is not protective against infection, and will not grow with the patient.

2023

80
Q

An 18-month-old female infant is evaluated because of oral asymmetry. A photograph is shown. This patient’s oral soft-tissue deficiency is best classified as which of the following Tessier cleft types?

A) No. 4
B) No. 5
C) No. 6
D) No. 7
E) No. 8

A

The correct response is Option D.

The Tessier classification provides a useful framework for defining rare soft tissue (shown in the illustration) and bony clefts of the face and craniofacial skeleton. Of these rare clefts, the Tessier No. 7 cleft is one of the more common and is often associated with craniofacial microsomia. Embryologically, it arises from failed fusion of the mandibular and maxillary prominences of the first branchial arch. The resultant soft-tissue defect can be quite variable, ranging from a subtle disruption of the oral commissure to a large open facial cleft extending from the oral cavity, across the cheek, and into the temporal region. Repair of the defect shown requires a multi-layered repair of the intraoral and commissure mucosa, all of the affected perioral musculature (especially the separated orbicularis oris muscle to establish oral continence), and the skin. Tessier No. 4 and No. 5 clefts extend from the upper lip to the lower eyelid, while the Tessier No. 6 cleft extends from the lateral lower eyelid into the ipsilateral cheek. Tessier No. 8 is a transverse cleft arising from the lateral canthal region.

2023

81
Q

A 3-month-old male infant is evaluated because of a midline congenital nasal mass. To determine the diagnosis and most appropriate management, which of the following imaging modalities would be most helpful in detecting associated brain abnormalities?

A) CT scan
B) Lateral cephalography
C) MRI
D) Skull x-ray study series
E) Ultrasonography

A

The correct response is Option C.

Midline congenital skull masses are most commonly dermoid cysts, encephaloceles, or gliomas.

To plan surgical resection of these masses, imaging is required to delineate diagnosis and extension. Concerns include intracranial extension through a patent skull base as well as associated intracranial anomalies.

Skull x-ray study series, ultrasonography, and lateral cephalography would not provide the detail needed to exclude intracranial extension or associated brain abnormalities. CT scanning is a useful adjunct in surgical planning and diagnosis of these patients, providing bony detail above and beyond what is possible with MRI.

However, for the most accurate diagnosis of structural brain abnormalities, MRI is the imaging modality of choice.

2023

82
Q

Which of the following best describes the order of permanent teeth eruption?

A) Lower premolar, upper incisor, upper canine, upper first molar
B) Upper canine, upper incisor, lower premolar, upper first molar
C) Upper first molar, upper incisor, lower premolar, upper canine
D) Upper first molar, upper incisor, upper canine, lower premolar
E) Upper incisor, upper canine, lower premolar, upper first molar

A

The correct response is Option C.

Mandibular incisors and canines erupt earlier than their maxillary counterparts. The first molar appears before the premolar and canine. The permanent canines are one of the last permanent teeth to erupt

2023

83
Q

The lateral cephalogram shown has a normal SNB angle of 80 degrees. These cephalometric findings are most consistent with which of the following?

A) Angle class II malocclusion
B) Mandibular prognathia
C) Maxillary retrusion
D) Overjet
E) Vertical maxillary excess

A

The correct response is Option C.

SNA, SNB, and ANB are standard measures taken on a lateral cephalogram to assess the absolute and comparative position of the mid face (point A, maxilla) and the lower jaw (point B, mandible) relative to the cranial base (SN, sella-nasion). The normal values for and relationship of these angular measurements are well-established and can vary with age, ethnicity, and other patient variables. In this cephalogram, SNB is given as normal and, thus, the mandibular position is not prognathic by definition. SNA is normally 2 to 3 degrees greater than SNB, but in this image, it is considerably less than SNB (negative ANB), indicating significant midfacial retrusion.

Overjet describes the relationship of the incisors wherein the top central incisors are anterior to the lower incisors in the sagittal plane; this is not seen in this image. The patient’s incisors are positioned edge-to-edge or at slightly negative overjet. Class II malocclusion is defined by the relationship of the maxillary and mandibular first molars, ie, the mesiobuccal cusp of the maxillary first molar rests anterior to the buccal grove of the first mandibular molar in occlusion. This relationship is not found in this image. Lastly, vertical maxillary excess, also called long face syndrome, is excess vertical maxillary length. These patients often present with a gummy smile and an under-projected (recessed) chin point due to autorotation of the mandible. The maxilla in this patient is vertically undergrown.

2023

84
Q

A 1-week-old female newborn who was born at term is evaluated because of respiratory distress. She has had apneic episodes and low oxygen saturation during sleep, which improves in the prone position. She also has had difficulty with weight gain. Which of the following is the most likely finding on physical examination?

A) Cleft palate
B) Eye colobomas
C) Low-set ears
D) Lower lip pits
E) Small tongue

A

The correct response is Option A.

The patient in the scenario described presents with airway obstruction that improves in prone position and poor weight gain. These findings are likely associated with Pierre Robin sequence, which includes micrognathia, glossoptosis, and airway compromise. Its incidence is around 1 in 10,000, and cleft palate is observed in approximately 80% of these patients. Lower lip pits are seen in Van der Woude syndrome. Small tongue, colobomas, and low-set ears can be associated with multiple syndromes, but they are not associated with Pierre Robin sequence.

2023

85
Q

A 5-month-old male infant is referred by neurosurgery for evaluation of a craniofacial dermoid cyst. Which of the following anatomic sites of a dermoid cyst is associated with the highest risk for intracranial extension?

A) Anterior fontanelle
B) Lateral brow
C) Lateral occiput
D) Medial canthus
E) Nasal tip

A

The correct response is Option A.

Dermoid cysts are a common congenital tumor of the head and neck. Imaging is often considered when evaluating these lesions. Based on a recent review of the literature, lesions located at the anterior fontanelle have the highest incidence of intracranial extension at 36.8%. Lateral brow, medial canthal, nasal tip, and occipital lesions were noted to have intracranial extension rates of 0.3%, 0%, 11.5%, and 16.3% respectively. Overland et al recommend imaging for any lesion located in the occipital, frontal, temporal, and midline nasal areas.

2023

86
Q

An otherwise healthy 21-month-old female infant is evaluated because of an insidious and progressive decline in head circumference percentile that began at age 6 months. Family history is significant for Crouzon syndrome. A head circumference chart and CT scan are shown. Which of the following best describes the findings on the CT scan?

A) Cloverleaf skull
B) Frontal lobe hypoplasia
C) Normal cranial shape
D) Patent cranial sutures
E) Turribrachycephaly

A

The correct response is Option C.

This patient has progressive postnatal pansynostosis (PPP), a relatively rare but important form of craniosynostosis that typically occurs in the first year of life. Unlike most other forms of craniosynostosis, affected patients have a normal head shape (often leading to a delay in diagnosis) and a slow unrelenting decline in head circumference percentile. This condition can occur in any child, but it is more common in patients with certain craniosynostosissyndromes, including but not limited to Crouzon syndrome. Because the sutures do not begin to fuse until after birth, the cranial shape typically remains normal (albeit smaller) and many of the common facial features seen with an associated craniosynostosis syndrome (eg, midfacial retrusion, exorbitism) may evolve in a delayed fashion once the sutures begin to close. The risk for intracranial pressure is quite high, greater than 75%, and cranial vault expansion is almost universally required. The age of onset of suture fusion correlates with the sudden decline in head circumference percentile, at age 6 to 8 months for this patient.

Patent cranial sutures would not adversely impact cranial growth unless there was a massive and acute cerebral event, which would impact the patient’s development. Unlike PPP, cloverleaf skull occurs as a result of prenatal multi-suture fusion and is clearly evident at birth. Frontal lobe atrophy can result in a small head, but the head circumference percentile typically stays consistent with growth. Turribrachycephaly results from prenatal fusion of both coronal sutures and, while often seen in patients with Crouzon syndrome, it would be clinically obvious at or around birth and would not typically result in a sudden decline in head circumference percentile as observed in this scenario.

2023

87
Q

A 6-month-old male infant is referred to the multidisciplinary craniofacial clinic because of a congenital full-thickness defect of the lower eyelid. This condition is most commonly associated with which of the following craniofacial syndromes?

A) 22q11 deletion syndrome
B) Möbius syndrome
C) Opitz syndrome
D) Treacher Collins syndrome
E) Van der Woude syndrome

A

The correct response is Option D.

The child described was born with a coloboma of the lower eyelid, most commonly associated with Treacher Collins syndrome. In addition to colobomas, Treacher Collins syndrome is characterized by downward slanting palpebral fissures, malar hypoplasia, conductive deafness, microtia and atresia, and cleft palate.

Möbius syndrome is characterized by cranial nerve dysfunction, including facial paralysis. Other symptoms include retrognathia, cleft palate, and extremity abnormalities.

The common features of 22q11 deletion syndrome include cleft palate, hypertelorism, and short philtrum, as well as cardiac and immunologic abnormalities.

Opitz syndrome includes cleft lip and palate in 50% of cases, hypertelorism, a prominent forehead, and low-set ears.

Van der Woude syndrome, the most common cause of syndromic clefting, is characterized by the presence of lip pits.

2023

88
Q

A 3-month-old male infant is evaluated because of an abnormal head shape. Physical examination shows biparietal narrowing with a cephalic index of 62 (N 76–81). There is a palpable ridge spanning anterior to posterior in the cranial midline. No ridge is palpable on the forehead. Which of the following is the most appropriate treatment?

A) Cranial molding orthotics
B) Fronto-orbital advancement
C) Monobloc advancement
D) Posterior vault distraction osteogenesis
E) Suture craniectomy and cranial spring application

A

The correct response is Option E.

The patient is presenting with sagittal craniosynostosis and scaphocephaly on examination. Although the surgical correction of sagittal craniosynostosis is controversial, the accepted options include spring-assisted surgery (SAS), strip craniectomy followed by cranial molding orthosis, and open vault remodeling cranioplasty. SAS entails removal of the synostotic suture, application of cranial springs, and subsequent hardware removal as a separate surgery.

Posterior vault distraction osteogenesis is typically reserved for multi-suture synostosis or in syndromic synostosis with concerns for increased intracranial pressure. Fronto-orbital advancement is typically performed for metopic and/or coronal synostosis and not performed at 3 months of age. Monobloc advancement is performed in syndromic coronal synostosis and indicated in those patients with concomitant severe obstructive sleep apnea and exorbitism. Cranial molding orthotics alone are not a first-line treatment.

2023

89
Q

An 18-year-old man comes to the emergency department because of a severe headache. He reports onset of sinonasal congestion, nasal discharge, and cough 6 weeks ago. During the past week, he also noted an impaired sense of smell, foul-tasting postnasal discharge, and an enlarging, painful mass on the left forehead. CT scan of the head shows opacification of the maxillary, ethmoid, sphenoid, and frontal sinuses. Additional findings can be observed in the representative CT scans shown. Which of the following is the most appropriate treatment?

A) Caldwell-Luc approach sinus drainage
B) Endoscopic sinus drainage and open cranialization
C) Incision and drainage of the left forehead mass
D) Long-term, narrow-spectrum antibiotic therapy for 6 weeks

A

The correct response is Option B.

The patient has developed a Pott puffy tumor in the setting of severe pansinusitis. Pott puffy tumor is not a true tumor, but rather osteomyelitis of the frontal bone with associated subperiosteal abscess resulting in forehead swelling. In addition to compromise of the outer table of the frontal sinus, the CT scan demonstrates inner table bone compromise with associated intracranial abscess. The most complete approach to treating this patient includes: 1) source control with drainage of all involved sinuses, including the frontal sinus; 2) drainage of the intracranial abscess with craniotomy; and 3) separation of the sinonasal cavity and the intracranial space with cranialization. Postoperative adjunctive therapy will likely include long term broad-spectrum antibiotic therapy since a mixed polymicrobial infection is most common. Isolated sinus drainage or sinus drainage and obliteration would not achieve the necessary goals to resolve the acute infection and avoid recurrence.

2023

90
Q

A 26-year-old woman is evaluated for class II malocclusion and obstructive sleep apnea refractory to conservative management. A lateral cephalogram shows an SNA angle of 77 degrees (N 82), SNB angle of 69 degrees (N 80), and an ANB angle of 9 degrees (N 2). Which of the following orthognathic movements is most appropriate to both treat this patient’s obstructive sleep apnea and improve her dentofacial profile?

Point A Point B Gnathion
A) Advanced advanced advanced
B) Advanced unchanged unchanged
C) Setback unchanged advanced
D) Unchanged advanced advanced
E) Unchanged setback advanced

A

The correct response is Option A.

This patient is presenting with class II malocclusion in the setting of both maxillary and mandibular hypoplasia based on her cephalometrics, although mandibular hypoplasia is comparatively more pronounced. Maxillomandibular advancement has been demonstrated to be an effective surgical treatment for obstructive sleep apnea. The procedure entails both LeFort I advancement as well as bilateral sagittal split mandibular advancement. To address her dentofacial concerns, the mandible would be advanced more than her maxilla. The movement would include point A advancement, point B advancement, and gnathion advancement. Any setback movements would likely negatively impact her obstructive sleep apnea.

2023

91
Q

A 6-year-old boy with a history of craniosynostosis is brought by his parents for routine follow-up evaluation. The patient underwent cranial vault remodeling in infancy. Annual ophthalmology follow-up is also planned. Which of the following types of craniosynostosis has the highest incidence of postoperative strabismus?

A ) Lambdoid
B ) Metopic
C ) Sagittal
D) Unilateral coronal

A

The correct response is Option D.

Serial ophthalmologic evaluation is necessary for patients previously treated for craniosynostosis to monitor for papilledema and other ocular pathology. Strabismus may be detected either preoperatively or postoperatively. Due to the significant asymmetry of orbital anatomy in patients with unilateral coronal craniosynostosis, the incidence of postoperative strabismus is higher in this population compared with other forms of single-suture craniosynostosis.

2023

92
Q

A pair of conjoined twins is evaluated to determine the surgical plan for soft-tissue coverage after their separation. The twins face one another and are conjoined from the sternum to the umbilical area. Which of the following terms best describes this type of conjoined twins?

A) Cephalopagus
B) Craniopagus
C) Omphalopagus
D) Parapagus
E) Thoracopagus

A

The correct response is Option E.

Conjoined twinning is an extremely rare phenomenon. They are monoamniotic and monochorionic twins that can be adjoined symmetrically, asymmetrically, ventrally, dorsally, laterally, or caudally. There is debate in the literature whether the phenomenon occurs because of fission of a single embryo at day 13 or fusion of two embryos between days 13 and 17 to create the single amnion/chorion.

In the United States, Willobee et al interrogated the Kids’ Inpatient Database and identified 240 conjoined twin pairs born between 1997 and 2012, for an incidence of 1 per 100,000 live births. Approximately 60% of those pairs died in the perinatal period. Only 28% were able to be separated in the neonatal period. The 2020 Great Ormond Street study by Frawley reported the following incidences:

  • Craniopagus – dorsal conjoined at the cranium – 5%
  • Cephalopagus – ventral fusion from vertex to umbilicus – 3.4%
  • Thoracopagus – ventral fusion of chest and thorax to umbilicus – 42%
  • Omphalopagus – ventral fusion of lower chest to umbilicus – 5.5%
  • Parapagus – lateral fusion of lower abdomen and pelvis – 14.5%

All studies emphasize the need for interdisciplinary management of these patients and stressed the use of imaging, model surgery, simulation surgery, and teamwork. The ethics of these cases can be very opaque with the high mortality for one or both twins, the intensive resources needed, and the highly publicized nature of these cases.

2023